Download as pdf or txt
Download as pdf or txt
You are on page 1of 64

PearsonRealize.

com

Selected Answers
Topic 1

Lesson 1-1 33. Yes; the quotient of a rational


number and an irrational number can
1. The sum, product, difference, or
be rewritten as a ratio with a rational
quotient of real numbers is a real
number in the numerator and an
number. Subsets of the real numbers
irrational number in the denominator,
are the set of rational numbers and
so the quotient will always be
the set of irrational numbers. The
irrational. 35. rational numbers
rational numbers include integers, the
37. no; no; no; yes; no; yes
integers include whole numbers, and
39. Part A Kimberly; she has the
the whole numbers include natural
highest free throw percentage.
numbers. 3. Rational numbers are a
Part B Martin; 49.5% is greater than
subset of all real numbers because all
920, or 45% and 0.448, or 44.8%
rational numbers are also real. Rational
Part C Kimberly, Corey, Martin
numbers are not a subset of the
Part D Kimberly; Kimberly makes
irrational numbers because no rational
about 54.8% of her shots, while Corey
numbers are irrational. 5. Yes. All the
only makes 52.8%, and Martin only
values ___ in D are also in D. 7. no 9. 0.6, makes 50.8%.
__
√2 __ 9
​​  3 ​​, ​​ ​  16  ​ ​​  11. a. rational b. rational
13. The student should have written ​​ Lesson 1-2
____ _____
√ 144 ​​ = 12. So, the correct order is √
​​ 144 ​​, 1. Use variables and operations to
​​  234
68.12, ___ 3
 ​​. 15. a. Sometimes true; model situations with algebraic
positive integers and zero are whole equations. Then use properties and
numbers, but negative integers are inverse operations to isolate the
not. b. Always true; natural numbers variable on one side of the equation.
are a subset of rational numbers. The other side of the equation is the
c. Never true; although integers and solution. 3. It removes the fractions
irrational numbers are both subsets of to make the equation easier to solve.
real numbers, they have no elements ​​  37
5. 2 7. ___ 4
 ​​  9. ​​ __ 50
21
 ​​, or about 2.38 miles;
in common. 17. rational, real Answers may vary. Sample: The
19. whole, integers, rational, real equation __ ​​  d
2
d
 ​​ + 2.25 + ​​ __
40
 ​​ = 3.5 models
21. integers,_rational, real this situation. Multiplying both sides by

​​  13 ​, ​ ​ __14 ​ ​​  25. D 27. A
23. 0.16, __ the LCD, 40, gives 20d + 90 + d = 140,
which simplifies to 21d = 50. 11. −3
29. Rational; can be expressed as a
ratio 31. Irrational; divisor is irrational

enVision™ Algebra 1 | 1 | Selected Answers


PearsonRealize.com

Selected Answers
Topic 1

13. 2.76 miles; first convert Parker’s the right hand side to get
running time to hours. Multiply 39 sec ​x − 2 = −x + 2​, and then continue
1 min
by _____
​​  60  ​​to convert 39 sec into 0.65 min. solving to find that the equation is true
sec
Then multiply 27.65 min by _____ 1 hr
​​ 60  ​​ to get for ​x = 2​. 5. x = 9 7. identity
sec 9. 4 games 11. The equation simplifies
0.46 hours. Use the Distance Formula
to ​0 = 0​; an infinite number; because
to find the number of miles he ran:
there are no variables in the simplified
d ≈ 6(0.46) = 2.76 miles. 15. Both;
equation and the result is a true
you can use the Division Property of
statement, any value will make the
Equality and divide both sides by __ ​​ 12 ​​
equation true. 13. Answers may vary.
(or multiply by 2). You can use the
Sample answer: ​x − 3 = x − 3​; It is an
Distributive Property to simplify the
identity, so it has an infinite number of
left side of the equation, ​​ __12 ​(2y + 4)​, solutions. 15. 8, no values, any value
which becomes ​y + 2​. 17. 5 19. 4 except −8 17. −12 19. ​​ ___34 ​​ 21. 7 23. −3
31 ​​, or 2​​ ___
7  ​​  27. ​​ ___
12 ​​, 9
21. ​−​7 23. 2 25. ​​ ___12 12 5 25. −10 27. −5 29. no solution
or 2​​ __ 96 ​​, or −3​​ ___
2 ​​  29. −​​ ___ 21 ​​  31. 4,300 31. ​−9​  33. identity 35. ​−2​  37. identity
5 25 25
39. 0.2 41. a. ​50 − 2d = 40 – 1.5d​;
33. 3.6 35. –80 37. 5 39. about 18
twenty 30-day periods b. $10
bonus payments per year; ​
43. Answers may vary. Sample answer: ​
1,000,000 = 6(20,000) + 8,000b​, where
p + 0.06p​ and ​1.06p;
b is the number of bonus payments
p + 0.06p = (1 + 0.06)p = 1.06p​ 
made over 6 years; so, ​b = 110​
45. a. Company A: ​500 + 25g​.
payments. So, the player is expected
110 Company B: ​200 + 30g​
to earn ​​ ___
6
 ​ ≈​ 18.3 bonus payments per b. Company A; Cost for Company
year on average. 41. 13.75; Because A is ​500 + 25(30) = $1,250​; Cost for
addition must be done first, the sum Company B is ​200 + 30(50) = $1,700​.
​3 + 8​should be in parentheses: ​ So, A is less expensive. 47. 48 ft 49. D
(3 + 8) × 5 ÷ 4​ 43. B, D 45. Part A
3 ​​ in. Lesson 1-4
18 rows of bricks Part B 60​​ __
8
1. Answers may vary. Sample: The
Lesson 1-3
resulting equation or formula can be
1. You can compare two relationships reused, so you need to solve only once
and find a quantity that makes another and then substitute different values to
quantity equal for each relationship. reuse the equation or formula.
3. Isabel did not consider the case 3. A literal equation is an equation
where the differences are equal to rule involving two or more variables.
zero. The correct approach would have A formula is a type of literal equation
been to apply Distributive Property to that states a relationship among
quantities. 5. x = y − 12 7. 43

enVision™ Algebra 1 | 2 | Selected Answers


PearsonRealize.com

Selected Answers
Topic 1

​c −a ​​,
9. ​x = _____ b 4; 12 11. Answers may 9. ​3.75x ≥ 1.5; x ≥ 0.4​  11. a. Answers
vary. Sample: The equation was not may vary. Sample: ​x + 3 > x + 4​; Since
solved for x because it appears on solving the inequality gives the false
4
each side of the solution; ​x = ​ ____ ​​  statement ​3 > 4​, there is no solution.
k+3
b. Answers may vary. Sample:
a ​​ 15. ​f = ​ ___
13. c = ​__ h  ​​  17. ​y = 4 − ___
​  2x ​​ 
b dg 3 ​x + 3 < x + 4​; Since solving the
1
19. ​b = ​ ____ ​​  21. x = ___ 6a 3V
​​  5 ​​  23. h = ​​ ___2 ​​  inequality gives the true statement
2ac ​πr​​  ​ ​3 < 4​, the solution is all real numbers.
ac F
​ r​​  2​
25. y = ​​  ________  ​​ 27. G = −​​  m ​​____ 13. The Distributive Property and the
a−b−c
7  ​​c + 40 b. c = ​​ ___ 30 ​​(T − 40) Subtraction Property of Inequality are
29. a. T = ​​ ___
30 7 applied in the same way, but when the
c. 210 chirps 31. a. ​​b​  1​​ = ___ 2A
​   ​− ​b​  2​​​  Division Property of Inequality is applied,
h
the inequality symbol is reversed.
b. 22 ft c. Answers may vary. Sample
answer: Solve ​22 − 2d = 6​; 8 ft 15. x > 6; 17. x ≤ –5;
33. ​c = ​___ ad ​​ 35. Part A ​h = 2r​; the
−5 0 5 10   −10 −5 0 5
b
height is 2 times the radius. Part B A
19. x ≥ –0.05;
can that meets the manufacturer’s
goals will have a label with area −0.06 −0.02 0.02
A = ​​πr​​  2​​.
21. x > 2;
Lesson 1-5
−2 0 2 4 6
1. The Multiplication and Division
Properties of Inequality are different 23. x ≥ 3; 25. x > 16;
from the Multiplication and Division −2 0 2 4 6 −10 0 10 20
Properties of Equality. When both sides
of an inequality are being multiplied 27. x > –4;
or divided by a negative quantity, −5 −4 −3 −2 −1 0 1 2
the inequality symbol is reversed.
3. Answers may vary. Sample: ​x > 3​ 29. B 31. C 33. ​x > 1​ 
and ​2x > 6​are equivalent; They have 35. ​x ≥ −7​ 37. ​x > −2.9​  39. ​x ≤ −1.25​
the same solution. 41. no solution 43. Fill in the circle
5. x < 12 to represent the solution ​x ≤ 12​.
45. ​55x − 15 < 50x; x < 3​; After 3 hours,
−5 0 5 10 15 Aisha will be in front of Luke.
7. x ≤ −1 47. ​4.25 + 2.25 + 4x ≤ 8; x ≤ 0.375​;
Charlie can buy up to 0.375 lb of potato
−2 −1 0 1 salad. 49. E

enVision™ Algebra 1 | 3 | Selected Answers


PearsonRealize.com

Selected Answers
Topic 1

Lesson 1-6 35. Part A ​500 ≤ 12x < 1000​;


¯6 ≤ x < 83.​ ¯
​41.​ ​ 3​​; A monthly donation
1. If the inequalities are joined by and,
from $41.67 to $83.33 will put Keenan
the solution includes only solutions of
in the Gold category.
both inequalities where they overlap.
Part B ​100 ≤ 50 + 3x < 500​;
If the inequalities are joined by or, the ¯
​16.​ ​ 6 ≤ x < 150​; Three donations from
solution includes the solutions of one
$16.67 to $150 will put Libby in the
inequality as well as the solutions of
Silver category.
the other inequality. 3. Answers may
Part C ​100 ≤ 12(50 − x) < 500​;
vary. Sample: A compound inequality ¯ 3 < x ≤ 41.​ ¯
​8.​ ​ 6​​; Paula can reduce her
combines or “mixes” two individual
monthly contribution by $8.34 to
inequalities. 5. ​x ≤ −4 or x ≥ −1​
$41.66 dollars per month.
7. ​x > 1 and x ≤ 4​;
Lesson 1-7
0 1 2 3 4 5

9. ​10x ≥ 12​and ​10x ≤ 15​; ​x ≥ 1.2​and 1. One equation or inequality


​x ≤ 1.5​; Nadeem will be riding between represents the case where the quantity
1.2 and 1.5 h, inclusive. 11. The student in the absolute value symbols is
graphed ​x ≥ 2​and ​x < 4​. The student positive, and the other equation or
should have graphed an open circle on inequality represents the case where
4 with the arrow pointing right. the quantity is negative. 3. The
13. a. The graph of x > a and ​x > b​ absolute value of a quantity represents
has an open circle on a with the arrow its distance from 0 on the number line,
pointing right, while the graph of and distance is always positive.
​x > a​or ​x > b​has an open circle on b 5. ​x = −2, x = 2​
with the arrow pointing right. b. The 7. ​x ≤ −1​or ​x ≥ 5​
graph of ​x > a​and ​x > b​has an open
circle on b with the arrow pointing −5 0 5 10
right, while the graph of ​x > a​or ​x > b​ 9. minimum: 2.1 hours; maximum:
has an open circle on a with the arrow 2.9 hours 11. ​n = 2​  13. No, an
pointing right. c. The graphs are not absolute value equation will not work
different; they are the same. 15. ​x < −2​ because the value of x would have
or ​x > 1​  17. ​x ≥ −0.5​and ​x < 0.25​  to be negative for the perimeter to
19. ​x > −4​and ​x < 2​  21. ​x > 8​  be 6 ft plus or minus 1.5 ft, and an
23. ​1 ≤ x < 3​  25. ​7 ≥ x ≥ 5​  27. ​x ≥ 10​ absolute value expression cannot have
and ​x ≤ 20​, or 10 ≤ x ≤ 20 29. x > 10 a negative value. 15. ​x = −3, x = 3​ 
and x < 20, or ​10 < x < 20​  31. Answers 17. ​x = −12, x = 12​  19. ​x = −6,
may vary. Sample: x < 20 + 20(30.5) or x = 14 ​  21. ​x = −18, x = 2​  23. no
x > 20 + 20(33.5); ​x < 630​or ​x > 690​; solution 25. ​| 5x – 10 | = 2.5​; minimum:
Cartons less than 630 ounces and 1.5 h; maximum: 2.5 h
greater than 690 ounces should be
opened for inspection. 33. A, C

enVision™ Algebra 1 | 4 | Selected Answers


PearsonRealize.com

Selected Answers
Topic 1

27. x ≤ –10 or x ≥ 10 27. The student added 5 instead of


subtracting 5 in the last step.
−20 −10 0 10 20 3 ​(b + 5)​
​a = ​ __
4
29. x ≤ –5 or x ≥ 5
​  34 ​ b + ​ ___
​a = __ 15 ​​
4
−10 −5 0 5 10
​  15
​a − ___ 4
3 ​ b​
 ​ = ​ __
4
31. x ≤ –7 or x ≥ 2
3(
​​ __ 4)
​  15
4 ​​ a − ___ ​  43 ​​(__
 ​ ​ = __ ​  34 ​ b)​
−8 −6 −4 −2 0 2 4 6 8
4 ​a − 5 = b​
​​ __
3
33. x ≤ –7 or x ≥ –1
ab − ​​
2 31. ​​v​  ​​​ = at + ​​v​  ​​​ = 14 ft/s
29. ​c = ​ ______
3b f i
−8 −6 −4 −2 0 2 4 6 8
33. ​x > 3​
35. B 37. A 39. The difference
−2.5 0 2.5 5 7.5
between the actual length of a case
x and 125 mm must, at most, be within 35. ​x < 25​
0.25 mm. The inequality | x − 125 | ≤ 0.25
0 10 20 30 40
can be used to represent this
acceptable range of lengths. Therefore,
37. ​45(x + 90) < 60x​
the range of lengths of cases that
should be removed can be represented ​45x + 4050 < 60x​
by the absolute value inequality ​4050 < 15x​
| x − 125 | > 0.25. ​270 < x​
41. ​| 2.50x − 25 | ≤ 0.50;​ ​9.8 ≤ x ≤ 10.2;​
Between 9.8 and 10.2 gallons will be After 270 minutes, or at 1:30 p.m., Yuki
pumped. 43. A will have typed more words than Neil.

Topic Review 39. ​x > 4​or ​x < 3​


0 2.5 5 7.5 10
1. Check students’ work. See Teacher’s
Edition for details. 3. compound 41. ​x > 4​or ​x < −1.5​
inequality 5. subset 7. literal equation __
9.__ Answers may vary. Sample: √ ​​ 2 ​​ and ​​ −5 −2.5 0 2.5 5
√8 ​​  11. real number, rational number,
43. She can buy 4 to 6 packages of
integer, ___ whole number, natural number
___ charms. 45. ​x = 2, x = −2​
√3 17
13. ​​ ​   ​ ​,​ ​0.​ ¯
___ 36​​, √
​​ 15 ​​  15. irrational; 28 47. ​−9 < x < 9​ 49. ​| x − 98.6 | ≥ 0.5​.
is not a perfect square, so the square | x – 98.6 | represents how far away
root of 28 is not rational. 17. ​x = 4​ the temperature is from 98.6°F. The
29 ​​ 21.
19. ​b = ​ ___ 1 ​t + t + 5.5 = 26.5​;
​1​ __ acceptable distance from the norm is
18 3 represented by 0.5. The solutions of the
Adult ticket = $12, child’s ticket = $9
inequality are the temperatures further
23. ​x = 0​  25. t = 36
from the norm than acceptable.

enVision™ Algebra 1 | 5 | Selected Answers


PearsonRealize.com

Selected Answers
Topic 2

Lesson 2-1 21. y


4
1. ​y = mx + b​. The slope of the line and 2
the y-intercept are given by the slope-
x
intercept form, so the line is easily −4 −2 O 2 4
graphed. 3. Emaan reverses vertical −2
and horizontal units in the slope when
plotting the second point, which should −4
be at (3, 6).
23. 4
y
5. y
2 2
x x
−2 O 2 4 6 −4 −2 O 2 4
−2

−4 −4

25. ​​y = 2x + 3​  27. ​​y = 3x − 3​ 


7. slope: –5; y-intercept: ___ ​​  –3 ​​ 
__ 4 4 ​  83 ​ x − ​ __
29. ​​y = 2x − 5​  31. ​​y = __ 2 ​​
9. y = –2x + 1 11. y = ​​  3 ​​x – 3 3
13. a. Answers may vary. Sample: Allie 33. ​​y = −1.5x + 4.5​ 
will move 3 units down and 4 units 35. Plan A: ​y = –60x + 720​, Plan B:
to the right to find the second point. ​y = −70x + 840​; The final payment for
Carolina will move 3 units up and four the two plans happens at the same
units to the left to find the second time. Even though Plan A involves a
point. b. Yes; they both find two points down payment, the higher weekly
on the same line using the equation. payment amount for Plan B causes
15. The y-intercept should be plotted the balance to decrease at a faster
at (0, −6), not (0, 6). The second point rate. 37. Luis’s ride: ​y = −7.5x + 15​;
would be at (4, −9). 17. ​b = 6​; ​n = 2.5​; Raul’s speed 5 mi/h; Raul’s ride:
​p = 4.5 ​y = −5x + 10​; both boys arrive at the
beach when ​y = 0​. 39. C
19. y
10
Lesson 2-2
5
1. The point-slope form of a line,
x
y ​−​​​y​  1​​​ = m(x ​−​​​x​  1​​​) reveals the slope m
−10 −5 O 5 10
−5 and one point (​​x​  1​​​, ​​y​  1​​​) on the line.
3. Denzel did not rewrite
−10
​y − 2 = ​ __23 ​(x + 3)​as ​y − 2 = ​ __32 ​(x − (−3))​
and used 3 instead of −3 for the
x-coordinate to find the first point.

enVision™ Algebra 1 | 6 | Selected Answers


PearsonRealize.com

Selected Answers
Topic 2

4 ​(x − 4) ​
5. y ​−​5 = ​−​3(x ​−​ 1) 7. ​y − 2 = −​ __
3 33. 2
y
4 ​(x − 1) ​9. y = ​−​5x + 1
or ​y − 6 = −​ __
3
3  ​(x + 5) o
11. a. ​y − ​4 ​= −​ ___ ​r
10 O 2 4 6 x
3  ​(x − 5) ​b. Answers may vary. −2
​y − 1 = −​ ___
10
Sample: __ ​​  52 ​​  c. Answers may vary. Sample:
The y-intercept is halfway between the −6
given points, so the y-intercept should
be about halfway between the two ​  32 ​(x​+ 4)​; ​y = __
35. ​y + 21 = __ ​​  32 ​​ ​  x − 15
y-coordinates. It will be between
5 ​​. d. ​y = – ​ ___ 3  ​x + __ 37. a. $608 b. $76 c. Both forms are
2 and 3, at about ​​ __ 2 10
​  52 ​​ useful for finding the amount she pays
13. a. ​​y − b = m(x − 0) b. The each week, which is the slope. Only
simplified form of ​y − b = m(x − 0)​is ​ slope-intercept form gives the original
y = mx + b​if you solve for y, so slope- amount borrowed as the y-intercept.
intercept and point-slope forms are ​​  97 ​​; (−4, 5); Answers may vary.
39. __
two ways of representing the same
Sample: (3, 14)
line. 15. ​y + 2 = −4(x − 2)​
17. ​y − 4 = ​ __ 2 ​(x + 1)​ 41. Part A ​y − 3.8 = 0.4(x − 1.5)​;
3 The slope is the rate of increase in
19. ​y − 7.5 = 1.5(x − 3.5)​
elevation: 0.4 m/sec.
21. ​y + 3 = (x − 1)​ Part B 3.2 m; slope-intercept form: ​
23. ​y + 7 = (x + 1)​or ​y + 4 = (x − 2)​  y = 0.4x + 3.2​; The y-intercept is
25. ​y − 12 = 5(x + 4)​ or ​y + 3 = 5(x + 7)​  the elevation where the railway
​  12 ​= −(x ​− 3)​or ​y − __
27. ​y + __ ​​  32 ​​ ​= −(x − 1) initially starts.
29. 4
y Lesson 2-3

2 1. The standard form of a linear


x equation is ​Ax + By = C​, where A and
−4 −2 O 2 4 B are not both equal to zero and A, B,
−2 and C are all integers. Standard form
makes it easy to identify intercepts.
The equation can be solved for either
y or x to find the x- or y-intercepts, and
31. 6
y
standard form makes it easy to see the
constraint by inspection. 3. Malcolm
4
did not use integers for all coefficients.
2 The equation should be ​3x + 2y = 8​.
x
−4 −2 O 2
−2

enVision™ Algebra 1 | 7 | Selected Answers


PearsonRealize.com

Selected Answers
Topic 2

5. 4 y 23. D 25. C 27. The graph of x = 2 is


a graph of the form ​Ax + By = C​ when
​B = 0​.
x
29. 4
y
O 2 4 6 8
−2
2
−4 x
−4 −2 O 2 4
7. 4
y

−4
2
x
O 2 6 31. y
x
−2 −4 −2 O 2 4
−4 −2

−4
9. ​3x + 2y = 12​, where x is pounds
−6
of green grapes and y is pounds of
red grapes. 11. Darren can solve the
second equation for y to verify that it
33. 4x − y = 18 35. x + 2y = −20
is identical to the first equation.
37. 2x + 8y = 16 39. –7x + 3y = –21
13. Point B: (−2, 0), point C: (0, 3);
41. 10 lb; 6 lb; ​3x + 5y = 30​, where x
Check students’ graphs. 15. x-intercept:
is pounds of cheddar and y is pounds
5; y-intercept: 2 17. x-intercept: 12;
of Swiss. 43. He can use the equation
y-intercept: 24
12x + 20y = 300, where x is number
19. y of hats and y is number of T-shirts.
4
Because hats and T-shirts cannot be
2 bought in fractions, solutions only
x correspond to points on the line
−4 −2 O 2 4 where x and y are whole numbers.
45. E
−4

21. 4
y

2
x
−4 −2 O 2 4
−2

−4

enVision™ Algebra 1 | 8 | Selected Answers


PearsonRealize.com

Selected Answers
Topic 2

Lesson 2-4 b. Sample: The artist can determine the


equation of each side of the figure. If
1. In slope-intercept form, the
opposite sides are parallel and adjacent
coefficient of x will determine the
sides are perpendicular, the figure is a
slope of the line. If the slopes are
rectangle. 31. a. y = 125x + 23 b. Yes;
the same, the lines will be parallel. If
the slopes of the lines are the same but
the slopes are opposite reciprocals,
the y-intercepts are different, so the
the lines will be perpendicular.
lines are parallel. c. Since the slopes
Otherwise the lines will be neither
are the same, Elijah and Aubrey deposit
parallel nor perpendicular. 3. Slopes
the same amount—$125—each week.
of parallel lines are the same; slopes
The y-intercepts are different which
of perpendicular lines are opposite
3 ​​x + 6
indicates that Elijah began with $72 in
reciprocals. 5. y = −​​ __
4 his account and Aubrey began with $23
1 ​​x + ​​ __
7. parallel 9. y = −​​ __ 1 ​​  in her account. 33. A
3 3
1 ​​
11. The slope of the line should be ​−​ __ Topic Review
4
1 ​(x − (−8))​
Corrected:​ y − 5 = −​ __ 1. Check students’ work. See Teacher’s
4
1 ​x − 2​ Edition for details. 3. standard form
y − 5 = −​ __
​ 4 5. slope-intercept form
1 ​x − 2 + 5​
y − 5 + 5 = −​ __
​ 4 7. 4
y
1 ​x + 3​
y − 5 = −​ __
​ 4 2
13. a. No, the slopes of adjacent sides x
must be negative reciprocals. The slope −4 −2 O 2 4
−2
of ¯
​​ AD​​ is −​​ __16 ​​, and the slope of ¯ BA​​
​​  is 5.
b. You could change D to (5, 1) and −4

C to (4, −4) so that two slopes would


9. ​y = 3x – 6​ 11. y = 50x + 250, $300
be ​−__ ​  15 ​​. 15. ​y = ​ __
1 ​x − 5 17. ​y = 2​ 
5 13. ​y – 5 = –3(x + 2)​
19. The slope of Line A is 2 and the 15. ​y – 4 = –0.5(x – 1.5)​
slope of Line B is ​−__ ​  12 ​​. The product of 17. ​A = 3, B = 5, 3x + 5y = 15​
the two slopes is −1. 21. ​y = −​ __ 4 ​x + 3​  19. ​y + 5x = 23​ 21. x-intercept: 24;
3
5 ​x + 8​  25. parallel 1 ​​ 25. ​y = ​ __
y-intercept: 8 23. ​– ​ __ 1 ​x + ​ __
1 ​​
23. ​y = −​ __ 4 3 2 2
27. perpendicular 29. a. Sample: 27. ​y – 6 = –2(x + 2)​ 29. parallel
​y = −​ __ 1 ​x + 3​The slope must be ​​−​​ __ 1 ​​.
2 2

enVision™ Algebra 1 | 9 | Selected Answers


PearsonRealize.com

Selected Answers
Topic 3

Lesson 3-1 11. Both a graph and a table show


that a relation is a function because
1. A function is a relation for which
you can see whether any of inputs
each input has exactly one output. It
have more than one output.
is important to know the domain and
13. D: {5, 8, 10, 12, 14}; R: {3, 6, 11};
range to define a function, because
The relation is a function because
only one element of the range can be
each input maps to exactly one
matched to an element in the domain
output. 15. domain: all positive real
for the relation to be a function.
numbers, range: 565 17. domain:
3. 2 points: 5 is in the domain and the
whole numbers; range: whole numbers
relation is not a function. 1 point:
between 0 and 2,000 19. function; not
5 is in the domain, and the relation
one-to-one 21. not a function
may be a function, depending on the
23. all whole numbers between 10 and
other points. no points: 5 is NOT in
20, inclusive 25. Answers may vary.
the domain, but the relation may be a
Sample: (A, half frown), (B, open oval),
function, depending on the other points.
(C, smile), (D, horizontal line), (F, frown)
5.  x −4 −2 −2 −1 3 3 Yes No
21. not a function
y −2 2 1 0 −2 1
a. Pilar ✓
23. allscored
whole 85 and studied forbetween
numbers 8 h. ❑ ❑
10 and
{(−4, −2), (−2, 2), (−2, 1), (−1, 0),
20, inclusive
(3, −2), (3, 1)} D: {−4, −2, −1, 3} b. Naida scored 97 and studied for 9 h. ✓ ❑ ❑
R: {−2, 0, 1, 2}; not a function 25. Answers may vary. Sample: (A, half
c. Alex scored 77 and studied for 4.5 h. ✓
❑ ❑
frown), (B, open oval), (C, smile),
7.  y
(D, horizontal
d. Damian scored 80line), (F, frown)
and studied ✓
for 7.5 h. ❑ ❑
4
27.
e. Dylan scored 90 and studied for 6 h. ❑ ✓

3

2
29. Part A Time (hours) Cost ($)
1
1 10
x
0 3 30
0 2 4 6 8
11 100
It is a function.
20 100
9. Yes; the domain is {3, 4, 5, 6, 7},
Part B Yes; cost is a function of time
which is only 5 values. The range
because each time value maps to one
contains 6 values, so at least one
and only one cost. Part C No; time is
domain value must be paired with
not a function of cost because the cost
more than one range value. The
value 100 maps to more than one time
relation is not a function.
value.

enVision™ Algebra 1 | 10 | Selected Answers


PearsonRealize.com

Selected Answers
Topic 3

Lesson 3-2 25. y


12
1. It is a function, has a graph that is or
8
falls along a straight line, and can be
written in the form ​y = mx + b​, where 4
m and b are constants. x
3. Answers may vary. Sample: A linear −4 O 4 8
function can be represented by words, −4
rules, tables, or graphs. A linear equation
is a first-order equation involving two 27. D: {–2, −1.5, −1, −0.5, 0, 0.5, 1, 1.5}
variables. 5. f(2) = 5; f(6) = 21 R: {−1.5, –1, −0.5, 0, 0.5, 1, 1.5, 2}.
29. a. ​f(t) = −1 + 1.5t​ 
7. y
8
8

Height Above Normal (ft)


6
6
4
4
2
2
x
−4 −2 O 2 4
O 2 4 6 8
1 ​​x + __
9. a. y = –​​ __ ​​  7 ​​ b. __
​​  1 ​​  Time (h)
3 3 3
5 b. No. The highest level, after 5 h, is
11. a. $8; ​​ __  ​​ lb; ​​ __ ​, ​  9  ​, __
7 __ 13
​  11 ​​, and ​​ __ ​​ 
16 16 16 16 16 ​f(5) = −1 + 1.5(5) = 6.5 ft​.
5
b. $8 × ​​ __
16
 ​ ​lb = $2.50; No, the cost of 31. a. D: ​{t ∣ 0 ≤ t ≤ 5.96}​,
the container does not vary. Answers R: ​{n ∣ 0 ≤ n ≤ 465}​
may vary. Sample: Whether the total n
weight is 7, 9, 11, or 13 ounces, the 400
Bottles Remaining

charge for the 5-oz container is always


5 300
$8 × ​​ __
16
 ​​ lb = $2.50. 13. f(5) = 21
15. f(5) = 20 17. f(5) = –9 200

19. y = 5x –1 21. y = –​​ __ 3 ​​x – 1 100


2
23. y 0
4 0 2 4 6 8t
Time (h)
2
x b. They would run out of water at
−4 −2 O 2 4 about 3:58 p.m. because the bottles run
−2 out about 5.96 hr after 10 a.m. They sell
78 bottles per hour, and the snack bar is
−4
open for 7.5 hours, so they should have
at least 78(7.5) = 585 water bottles at
the start of the next business day. 33. E.
enVision™ Algebra 1 | 11 | Selected Answers
PearsonRealize.com

Selected Answers
Topic 3

Lesson 3-3 Lesson 3-4


1. Identifying the transformations of 1. An arithmetic sequence is a linear
a linear function helps you determine pattern. The graph of an arithmetic
how much to shift the graph of the line sequence is a series of points that lie
horizontally or vertically and whether along a line that can be described by a
the slope and y-intercept of the graph linear function. 3. Answers may vary.
will change. Sample: A recursive formula is good to
3. When a number is added to the use if you want to describe the pattern
equation ​y = f(x)​, the line is shifted of a sequence or if you want to find
vertically, but when a number is added the next term in a sequence and you
to x, the line is shifted horizontally. know the previous term. 5. yes
5. shifted 3 units to the left ​​7. The 7. ​​a​  n​​​ = ​​a​  n−1​​​ + 4 and ​​a​  1​​​ = 81
slope and y-intercept are scaled by 9. ​​a​  n​​​ = 5n; $55 11. Answers may vary.
a factor of 4.​  ​9. The graph is shifted Sample: The domain of an arithmetic
up 75 units. 11. Translations do not sequence is natural numbers. The
change the slope of the line, so the domain of a linear function is all real
original line and the translated line are numbers. 13. Explicit formula, you can
always parallel. 13. h(x) = 2x − 6; use the explicit formula to find the
h(x) is equal to f(4x − 6). 15. shifted up value of the term based on the term
8 units 17. shifted left 10 units number. To use the explicit formula,
19. horizontal stretch by a factor of 0.1 you would need only to substitute 500
21. horizontal compression by a factor for n. If you used a recursive formula to
of 2 23. slope and y-intercept scaled by find the value of the term ​​a​  500​​​ when
a factor of 3 25. slope and y-intercept only knowing ​​a​  1​​​, then you would
scaled by a factor of __ ​​ 16 ​​  27. slope is the have to add the common difference
same, y-intercept is shifted down 499 times; Yes, if you knew ​​a​  499​​​, then
3 units 29. k ​=​3; vertical translation up all you need to do to find ​​a​  500​​​ using
31. a. f(x) = 40x + 100; g(x) = 80x + 100 a recursive formula is substitute for
b. The slope of g is twice the slope of f, the value of ​​a​  499​​​. 15. No; Answers
and the y-intercept of g is the same as may vary. Sample: Term numbers are
the y-intercept of f. 33. a. downward positive integers. Because 2.5 is not
shift b. shift to the right c. positive a positive integer, it is not a term
slope and y-intercept scaled by a factor number in an arithmetic sequence.
of k d. positive slope 35. B Substituting this value into an explicit
formula will not give you a term in the
sequence. 17. yes; 14 19. no
21. no 23. yes; –7 25. ​​a​  n​​ = ​a​  n – 1​​ + 7​
and ​​a​  1​​ = 12​; ​​a​  n​​ = 5 + 7n​ 

enVision™ Algebra 1 | 12 | Selected Answers


PearsonRealize.com

Selected Answers
Topic 3

27. ​​a​  n​​ = ​a​  n – 1​​ – 5​and ​​a​  1​​ = 62​; 17. positive correlation
​​a​  n​​ = 67 – 5n​  29. ​​a​  n​​ = ​a​  n – 1​​ – 4​and y
​​a​  2​​ = 12​; ​​a​  n​​ = 20 – 4n​  10
31. ​​a​  n​​ = −7 + 15n​  33. ​​a​  n​​ = 1 − 2n​ 
35.​ ​a​  n​​ = 11 + n​ 
37. ​​a​  n​​​ = ​​a​  n−1​​​ + 8; ​​a​  1​​​ = 18 5
39. ​​a​  n​​ = ​a​  n – 1​​ + 12​; ​​a​  1​​ = –17​ 
41. ​​a​  n​​ = ​a​  n−1​​ − 3; ​a​  1​​ = __ ​  1 ​​ 
2
43. ​​a​  n​​ = –6 + 7n​; the 50th white key x
from the left 45. 5,500; the level O 5 10
completed; ​​a​  n​​ = 2,250 + 3,250n​;
​​a​  n​​ = ​a​  n − 1​​ + 3,250​and ​​a​  1​​ = 5,500​  47. C 19. no association
y
Lesson 3-5 10

1. A scatter plot helps to show if there 8


is a relationship between the two data 6
variables. 3. y-values increase 5. Select
two points on the trend line. Use these 4
two points to find the slope of the line. 2
Then use point-slope formula with one x
of the points on the trend line and the O 2 4 6 8 10
slope to find an equation for the trend
21. ​y = −1.25x + 13​
line. 7. negative 9. No; you need to
y
select two points that lie on the trend 10
line. These points may or may not be
given data points. 11. Although y
is increasing, x is decreasing, so the
5
data show negative correlation.
13. Answers may vary. Sample: A data
set where, as x-values increase, y-values
stay constant. While these data would x
show neither a positive nor negative O 5 10
association, a constant trend line would
23. Positive; because the height of a
be valid. 15. negative
plant increases as the number of days
since germination increases, the data
would show positive association.

enVision™ Algebra 1 | 13 | Selected Answers


PearsonRealize.com

Selected Answers
Topic 3

25. Answers may vary. Sample: 21. The model is a good fit.
​y = 0.2x + 0.5​; the increase in feet y
1
of maximum recommended viewing
distance per increase in inch of screen
size 27. B
x
Lesson 3-6 O 10 20 30 40
1. Look at the correlation coefficient
to tell how well the line of best fit
models the data. 3. A correlation −1
coefficient of −0.93 is close to −1, so it
indicates strong negative correlation, 23. No, there will be exceptions.
not weak correlation. Weak correlation 25. The slope represents the change in
is indicated by a correlation coefficient trillions of vehicle-miles traveled each
that is close to 0. 5. y = 0.39x + 3.1 year, and the y-intercept represents
7. about 93 customers 9. The student the trillions of vehicle-miles traveled in
switched the order of the data. The 1975; 3.138 trillion vehicle-miles;
student should enter x data in L1 and y 3.858 trillion vehicle-miles 27. a. strong
data in L2. The line of best fit is b. neither c. strong d. weak
​y = 3.94x − 6.86​. 11. Answers may 29. Part A ​y = −2.9x + 7465.2​; ​r = −0.83​ 
vary. Sample: In general, the slope of Part B ​y = −6.42x + 14,347.64​;
a linear model represents the how the ​r = −0.94​  Part C The data from 1940–
two types of data change in relation to 2010 have a stronger correlation than the
one another. The y-intercept of a linear data from 1940–2010; Answers may vary.
model represents the starting value, Sample: The correlation coefficient of the
or when the independent variable data from 1940–2010 is closer to ​−​1 than
has a value of 0. 13. Tavon’s model the correlation coefficient of the data
is better than Arthur’s because the from 1940–1980, so the data from 1940–
points on Tavon’s residual plot are, on 2010 have a stronger linear relationship
average, closer to the origin than the than the data from 1940–1980.
points on Arthur’s residual plot. Topic Review
15. ​y = −5.73x + 197.2​; 88.33
17. strong positive correlation 1. Check students’ work. See Teacher’s
19. weak negative correlation Edition for details. 3. residual

enVision™ Algebra 1 | 14 | Selected Answers


PearsonRealize.com

Selected Answers
Topic 3

5. line of best fit 7. domain: {0, 2, 4, 5}; 33. y


50
range: {1, 3, 4}; The relation is a
function because each input has 40
only one output. 9. All real numbers 30
would not be a reasonable domain
because the number of drinks cannot 20
be negative. 11. The domain should 10
be between zero and a reasonable x
altitude. 13. {−9, −5, −1, 3, 7} 0
0 4 8 12 16
15. The page rate is $75 per page; Her
Answers may vary. Sample:
setup fee is $35. 17. It shifts it down 13 1 ​​  35. ​y = 1.8x − 27.2; 17.8​ 
​y = −​ ___ ​x + 44​ __
2 units. 19. The graph of ​f​ is translated 9 3
3 units to the right. 21. Each value 37. Interpolation and extrapolation are
doubles. 23. No; 48 ​−​45 ​=​3 and both methods of making predictions
45 ​−​41 ​=​4, so there is not a common about values in a data set. However,
difference. 25. ​​a​  1​​​ = 2; ​​a​  n​​ = ​a​  n − 1​​ +​ ​4​  interpolation makes a prediction within
27. ​​a​  n​​ = 8 + 5 (n − 1) or ​a​  n​​ = 5n + 3​; the known data set while extrapolation
78 29. no association 31. The line of makes a prediction outside the known
best fit should be positioned so that data set.
the average distance from the data
points to the line is as minimal as
possible.

enVision™ Algebra 1 | 15 | Selected Answers


PearsonRealize.com

Selected Answers
Topic 4

Lesson 4-1 17. infinitely many solutions


19. a. 10 weeks b. $250 each
1. Graph the two linear equations, and
21. exactly (0.2, 3.8)
wherever the graphs intersect is the
23. approximately (−4.615, 7.462)
solution of the system. 3. The point of
25. a. ​y = 15 + 32x​
intersection lies on both lines, so the
​y = 35x​
x- and y-coordinate of the point satisfy
The solution is (5, 175).
both equations in the system.
b. The solution means that for 5 jackets,
5. y the cost would be $175 for either
company. c. Gabriela should buy the
2 jackets from Anastasia’s Monograms.
(−2, 1) x The graph shows that the cost for
−4 −2 O 2 4 jackets from Monograms Unlimited
−2 overtakes the cost for jackets from
−4 Anastasia’s Monograms for orders
greater than 5 jackets. 27. two, (0, 2)
7. 1 gallon; Brand B paint and a grid ​  3 ​x + 1​
y = __
29. Part A a. ​
2
​  5 ​x − 4​ 
9. Answers may vary. Sample: ​y = __
2
11. The student did not rewrite both ​  −9 ​x + 13​
y​ = ___
2
equations in the same form before
drawing the conclusion that the ​  3 ​x + 1​
b. ​y = __
2
coefficients were the same. The system
​  −1 ​x − 3​
y​ = ___
has exactly one solution: (0, 9). 2
13. y
4 ​  −1 ​x − 3​
c. ​y = ___
2
2
​  −9 ​x + 13​
​y = ___
x 2
−4 −2 O 4 Part B Answers may vary. Sample:
−2 (2, 4), (−2, −2), and (3, −2)
−4 (1, −4) Part C The system y = __ ​  32 ​x + 1
​​ ​ ​​
y = −6x + 16
15. 4
y produces the solution (2, 4).
3
2 The system y = ​ __2 ​x + 1
​​  ​​​
x y = −2
−4 −2 O 2 4 produces the solution (−2, −2).
−2
(−2, −3) The system y = −2
−4 ​​ ​ ​​
y = −6x + 16
produces the solution (3, −2).

enVision™ Algebra 1 | 16 | Selected Answers


PearsonRealize.com

Selected Answers
Topic 4

Lesson 4-2 Lesson 4-3


1. Solve one equation for one of the 1. Because the Addition Property of
variables. Substitute the expression for Equality says that if you add the same
that variable into the other equation quantity to each side of an equation,
and solve for the first variable. Then then the result is an equivalent
substitute that value into either equation. 3. If the two systems can
equation to solve for the second be multiplied by different constants
variable. 3. Only the points on the line to become the same system, the two
are solutions to the system. Points not systems are equivalent. 5. (−2, −3)
on the line are not solutions. 5. (2, 4) 7. (−3, −1) 9. 37 small prints and 15
7. (−4, 5) large prints 11. In a system with no
9. x + y = 70 solution, the values of A and B are the
65x + 15y = 2,400 same when none of the numbers in an
43 Limited Edition and 27 Pro NSL equation have a common factor. The
soccer balls were sold. 11. The slopes value of C cannot be the same in both
are equal, but the y-intercepts are not equations. In a system with infinitely
equal. 13. If the constants are the many solutions, the values of A, B,
same, the lines must be the same, so and C will be the same when none of
there is an infinite number of solutions. the numbers in an equation have a
If the constants are different, there are common factor.
two lines, which means the lines are 13. The student forgot to multiply the
parallel. 15. 28º and 62º 17. (7, 10) right side of the equation by ​−​1.
19. (2, −3) 21. (2, 2)
23. The correct answer is
y
6
2x – y = –1
4 (1, 4)
–1(x – y) = (–1) • (–4)
2
x
​2x − y = −1​
−2 O 2 4 ​−x + y = 4​
25. Answers may vary. Sample: ​x = 3​
graphing, because the solution
has a precise answer 27. infinitely
​2(3) − y = −1​
many solutions 29. infinitely many
solutions 31. Richard is 22 and Teo is 9. ​6 − y = −1​
33. They scored eight 3-point shots
​−y = −7​
and thirteen 2-point shots.
35. Site A: 376 ​y = 7​
Site B: 324 The solution is (3, 7).
37. A
enVision™ Algebra 1 | 17 | Selected Answers
PearsonRealize.com

Selected Answers
Topic 4

15. (3, −1) 17. (4, −2) 19. (−4, −4) 35. 5 37. 3, −2
21. (−5, −2) 23. No; there is no number​ 39. Part A Concessions Unlimited:
3x − 9y = 5​can be multiplied by to 4x + 3y = 12.5
equal ​6x − 9y = 10​. 25. Yes; ​10x + 6y = 38​ 2x + 5y = 15
is equivalent to two times ​5x + 3y = 19,​
and ​10x + 20y = 100​is equivalent to Snacks To Go:
five times ​2x + 4y = 20​. 3x + 3y = 10.5
4x + 2y = 10
27. Let x = cost of a hat
Let y = cost of a shirt Part B (1.25, 2.5), (1.5, 2); the solutions
8x + 3y = 65 represent the costs per granola bar and
2x + 2y = 30 per drink for each concession stand.
Cost of a hat: $4 Part C Answers may vary. Sample:
Cost of a T-shirt: $11 Granola bars cost $1 and drinks
cost $1.50; For one granola bar and one
29. Substitution; Substitute y − 4 for x
drink, the equation would be
in the first equation and solve; (3, 7)
​x + y = 2.5.​For one granola bar and
31. Elimination; Multiply the first
two drinks, the equation would be
equation by 4 and the second equation
x + 2y = 4. So a system of equations
by 3. Then add equations to eliminate
would be as follows:
the y-terms; (4, –2)
x + y = 2.5
33. a. Graphing is most efficient if the x + 2y = 4
equations in the system are in slope-
Part D The solution represents the
intercept form or if there is an integer
number of granola bars and drinks that
y-intercept. If the lines intersect,
should be purchased so the overall cost
graphing is best when both coordinates
is the same at both stands.
of the solution are integers.
b. Substitution is most efficient when Lesson 4-4
one equation or both equations
has been solved for either x or y. 1. The shaded region and any point on
Substitution is a more efficient the graph line, if it is solid, contain all
method when the solution has rational possible solutions to the inequality.
numbers. c. Elimination is most 3. an ordered pair (x, y) 5. no
efficient when either the coefficients 7. y
of the x-terms or y-terms in the
2
equations are opposites and can easily
x
be eliminated. Elimination is also the −2 O 2
best method when all variables have
a coefficient. Elimination is also more
efficient when the solution has rational
numbers.

enVision™ Algebra 1 | 18 | Selected Answers


PearsonRealize.com

Selected Answers
Topic 4

9. ​y ≤ −x + 2​  11. The student thought 23. y


that ​1 ≤ 1​was not a true statement. 2
The statement is true because the x
symbol is greater than or equal to, and −2 O 2
1 is equal to 1. So, (1, 1) is a solution −2
of the inequality. 13. Answers may
vary. Sample: ​y ≥ __ ​  23
​  45 ​x + __ 5
 ​ ​  15. When an
inequality is solved for y, the border ​  1 ​x + 2​ 27. ​y ≥ −​ __
25. ​y < __ 1 ​x + 4​
3 4
line is determined by the equation
that is formed by substituting an 29. a. 25x + 60y ≤ 2,500;
equal sign for the inequality sign. y
Then, use the inequality symbol to 40
determine whether y-values should
20
be greater than the border line or
x
less than the border line. y-values O 40 80
greater than the border line are
found above the line and y-values less
than the border line are found below b. 25x + 60y ≤ 2,320;
the line. y
17. y x ​​ 40
O 2 4 6
20
−2
x
−4 O 40 80

−6
c. The graph from part (b) is the graph
from part (a) shifted down by 3. The
19. y
y-intercept shifts from ​41​ __23 ​​ to ​38​ __32 ​​.
2
Also, the x-intercept shifts from 100 to
x
92.8. There are fewer possible solutions
−2 O 2
in the first quadrant when the mover
−2
rides than when the mover does
not ride.
21. y
2
x
−2 O 2
−2

enVision™ Algebra 1 | 19 | Selected Answers


PearsonRealize.com

Selected Answers
Topic 4

31. yes; yes; no; yes 7. y


4
33. Part A ​3.6x + 4y ≤ 115​
Part B y ; x
20 −4 −2 O 2 4
−2
10
−4
O 10 20 x
9. ​y < 4​
​ x > −2​
As the number of photos stored
increases, the number of songs 11. The student used the wrong
that can be stored decreases. inequality symbol in writing the second
Part C No; although you could inequality. Instead of ​y < –2x + 2​, it
consider adding negative photos or should be ​y > –2x + 2.​  13. ​x ≥ 0​and
songs as deleting a photo or song, the ​y ≥ 0​; Answers may vary. Sample: These
graph would not provide a good model inequalities indicate that all values of
of this because it allows only deleting x and y must be 0 or positive; thus, they
or adding for each type of item. The limit the solutions to the first quadrant.
graph does not include the possibility 15. Yes; Answers may vary. Sample:
of deleting some photos and then ​y ≥ 1​, ​y ≤ 3​,​ x ≥ 1​, ​x ≤ 3​
adding additional photos or deleting 17. y
some songs and then adding additional 4
songs. 2
x
Lesson 4-5
−4 −2 O 2 4
1. The overlapping shaded region −2
contains all possible solutions to the
−4
system of inequalities. 3. at least 2
5. ​y = −3x + 4​and ​y = 8x + 1​
19. y
4

2
x
−4 O 2 4
−2

−4

enVision™ Algebra 1 | 20 | Selected Answers


PearsonRealize.com

Selected Answers
Topic 4

21. y 33. ​x + y ≤ 80​and ​10x + 15y ≥ 1,000​;


2 ​ 2 ​​ hours
0 hours babysitting and ​66 __
3
x providing tech support
−2 O 2
−2 y
80

60
23. 2
y
40
x
−2 O 2 20
−2
O 20 40 60 80 x

35. Neither graph is correct. The


25. y boundary lines for both inequalities
2 in the system have positive slopes, but
x both graphs show one boundary line
−2 O 2 with a negative slope. 37. C
−2
Topic Review
1. Check students’ work. See Teacher’s
27. y Edition for details. 3. linear inequality
x
in two variables 5. solution of a
−4 −2 O 2 linear inequality in two variables
−2 7. (−2.5, 0.4) 9. A+ Food:
y = 35x + 75, Super Cater: y = 38x;
−4 The solution is (25, 950). This
indicates that it costs $950 to buy
1 ​x – 2​
29. ​y ≥ –​ __ 25 gift baskets from both companies.
4
​ y < 2x + 4​ Kiyo should use A+ Food if she
expects 28 guests. 11. ( )
5 ​ ​​
​  11 ​, ​ __
​​ ___
31. ​y ≤ –x + 4​ 4 2
​ y ≥ 4x – 9​ 13. approximately (3.355, 0.388)
​ x < 2​ 15. infinitely many solutions 17. 152
boxed action figures, 94 collector pins
19. ​​(___
23 23 )
​  18 ​, – ​ ___
17 ​ ​​  21. No; there is no

number you could multiply


​3x − 4y = −6​by to get ​6x − 8y = 12​.
23. pens = $3.95/pack,
paper = $4.95/pack 25. no

enVision™ Algebra 1 | 21 | Selected Answers


PearsonRealize.com

Selected Answers
Topic 4

27. y 31. y
4

2
x x
−4 −2 O 4 −4 O 2 4
−2 −2

−4 −4

29. A
 nswers may vary. Sample: 33. x ≥ 0, y ≥ 0
y<x+4

enVision™ Algebra 1 | 22 | Selected Answers


PearsonRealize.com

Selected Answers
Topic 5

Lesson 5-1 13. –1 and 1, because the slopes of the


two sides of the graph of g(x) = a|x| are
1. The absolute value function is the
−a and a. The product of the slopes
function f(x) = | x |. Its graph is V-shaped
must be −1 if there is a right angle at
and symmetric about a vertical line of
the vertex, so ​−​a​​  2​​ = −1, so a = −1 or
symmetry (the y-axis) containing the
a = 1.
vertex (the origin). 3. Answers may
vary. Sample: (−1, 16); a = 16 15. a. y
8
5. Domain: all real numbers;
Range: ​y ≥ 0​ 6

7. y 4
6
2
4
x
2 −4 −2 O 2 4
x
−4 −2 O 2 4
b. 2 c. The rate of change is equal to
−2
the coefficient of the absolute value
term. 17. no 19. no 21. no
9. m = 0.6 23. y
8
11. Answers may vary. Sample:
• Both have linear sections. 6
• Both have a domain of all real 4
numbers.
• Both have the same values when ​ 2
x ≥ 0​. x
−4 −2 O 2 4
• The graph of f(x) = |x| is symmetric
about the y-axis, but the graph of ​  omain: all real numbers;
D
f(x) = x​is not. Range: ​y ≥ 0​
• The graphs have opposite values
25. y
when x​ < 0​. 3
• The range of f(x) = |x| is ​y ≥ 0​, 2
but the range of f(x) = x is all real
numbers. 1
• Both have an x-intercept and x
y-intercept at the origin. −4 −2 O 2 4
• When ​x < 0​the graph of −1
f(x) = |x| is decreasing, but the  omain: all real numbers;
D
graph of f(x) = x is increasing. Range: y ≥ 0

enVision™ Algebra 1 | 23 | Selected Answers


PearsonRealize.com

Selected Answers
Topic 5

{−1.5x,  x ≥ 0
1.5x,    x < 0
​​  4 ​​ ft/s 29. g(x) = 3|x| 31. through
27. __ 19. f(x) = ​​   
​  ​​​​
3
A and B: ​f(x) = __ ​  14 ​∣x∣​; through C and D: ​
21. y
g(x) = __ ​  32 ​∣x∣​ 33. 18 ft 35. D 8

4
Lesson 5-2
x
1. The pieces of a piecewise-defined O 8
function are assigned to intervals of −4
the domain with each interval having a increasing: x > 6; decreasing: x ≤ 6
different function rule. 3. The value 23. Yes; if she sends 200 messages it
−3 is included in both intervals. will cost $16.00 and if she sends 201
{−5x, x < 0
5x,    x ≥ 0 messages it will cost $12.06.
5. ​f(x) = ​ ​ ​​​​
​  15 ​x − __
__ ​  75 ​,   x ≤ 2
{__​  21 ​x + 1,   x > 2
7. 4
y 25. f(x) = ​​ ​  ​​​​

{82x + 62,    x > 5


70x + 122, x ≤ 5
x 27. a. f(x) = ​​   
​ ​​​​
−4 −2 O 2 4
−2 b. ​f(8) = 718​  c. f(8) − 122 represents
the amount of money Reagan has
−4
deposited in his savings account from
his summer job for the first eight
{    x,            x ≥ 1
−0.5x + 1, x < 1
9. f(x) = ​​ ​  ​​​​ weeks. 29. all real numbers; all real
numbers; three; 4
11. Answers may vary. Sample: The 31. Part A Vic charges 9.5(10) = $95.
intervals that are the domains of the Sue charges 15 + 9(10) = $105.
pieces make up the entire domain of
Part B Sue: 10; Vic: 9.622; Over
the piecewise-defined function with no
the given interval, Sue’s average

{–2x + 6, x ≥ 1
​  12 ​x + __
__ ​  72 ​,   x < 1 babysitting rate is $10 per hour and
overlap. 13. f(x) = ​​   ​  ​​​;​
Vic’s average rate is about $9.63 per
The x-intercepts are –7 and 3, and the hour.
​​  72 ​​. 15. a. The y-intercept
y-intercept is __
Lesson 5-3
is included in the piece of the function
that crosses the y-axis, which would 1. Step functions are piecewise-defined
be the piece for ​x ≤ 1​. So, find the functions that have graphs with an
y-intercept for the function rule for infinite number of pieces consisting of
that piece. b. Identify the piece where ​ horizontal line segments. 3. The values
x = 0​and find the y-intercept of the 10 and 20 are each included in two
function rule for that piece. intervals. 5. 6 7. 9

{−x,  x ≥ 0
x,    x < 0
17. f(x) = ​​ ​ ​​​​

enVision™ Algebra 1 | 24 | Selected Answers


PearsonRealize.com

Selected Answers
Topic 5

9. 10 y
⎧1, 0 < x ≤ 50      
8 ⎪2, 50 < x ≤ 100  
27. a. f(x) = ⎨

6 ​ 100 < x ≤ 150​​​​​
​​ 3,
  
4, 150 < x ≤ 200
⎩5, 200 < x ≤ 240
4

2
b. Answers may vary. Sample: You
x
0 need to assume that there are no
0 2 4 6 8
other people going on this trip by
11. The x-values of the pieces of a step taking buses other than seniors.
function make up the domain; this is c. 0.05; 0 d. The rate of change 0.05
because every element in the domain has between 40 and 60 students tells you
to be in one of the pieces. 13. Answers that the number of buses will change
may vary. Sample: At ​x = 1​, there is a for somewhere between 40 and
closed point at (1, 3) and an open point 60 students. The rate of change 0 tells
at (1, 4), so you have to determine that you that the number of buses does not
the value of the function at ​x = 1​is 3. change when the number of students
When finding the value at ​x = 1​on a changes from 60 to 80.
line, there is only one point to consider


at ​x = 1​. 15. 1 17. −5 19. 7 21. 23 29. a. f(x) = –50x + 350;
23. y ⎧ 350, x = 0      
10
300, 0 < x ≤ 1
250, 1 < x ≤ 2


8
200, 2 < x ≤ 3
6 f(x) = ⎨ ​ 
​​    ​​​​
150, 3 < x ≤ 4
4
100, 4 < x ≤ 5
2 50, 5 < x ≤ 6  
x ⎩0, 6 < x ≤ 7    
0
0 2 4 6 8
b. The first graph and function
f(x) = ceiling(x) + 4 represent the situation as if money
was being withdrawn constantly from
25. y
12 Mia’s account. The second graph
10
and function represent money being
withdrawn once each week. c. The
8 first function is easier to use for solving
6
equations. The second function more
accurately represents the situation.
4 31. D
2
x
0
0 2 4 6 8 10
enVision™ Algebra 1 | 25 | Selected Answers
PearsonRealize.com

Selected Answers
Topic 5

Lesson 5-4 17. (0, 1)


y
1. Answers may vary. Sample: The
values of the constants affect the 4
graphs of piecewise-defined functions 2
in the same way they affect the graphs x
of linear functions. 3. No; ​f(x) = ∣4x − 1∣ −2 O 2
= ​ 4​(x − __
| ​  14 ​)​  ​ = 4​|x − __
| ​  14 ​|​ ≠ 4∣x −1∣​ 
5. (0, 2.5) 19. (1, 0)
y y

4 4

2 2
x x
−2 O 2 −2 O 2

7. (2, 4) 21. (0.5, 0.5)


y y
6
4
4
2
2 x
x −2 O 2
O 2 4

​​  12 ​​​​|x − 2|​​ − 1


9. f(x) = __ 23. T
 he graph of g is a vertical stretch
11. a. f(x) = –2(x + 1) – 7 of the graph of f by a factor of 4
that is then reflected across the
b. f(x) = a(x – h) + k and
x-axis and translated 2 units right
f(x) = –a(x – h) + k and 1 unit down.
13. The student did not describe the y
reflection across the x-axis, incorrectly O x
translated the vertex to the right 2 4
instead of to the left, and implied that −2
only one point was translated. The −4
graph of ​y = −0.5∣x + 1∣ + 3​compresses
the graph of ​y = ∣x∣​, reflects the graph
across the x-axis, and translates the
graph left 1 unit and up 3 units.
15. The graph of ​​Y​  2​​​ = INT(x) − 2
is a translation 3 units down of the
graph of ​​Y​  1​​​ = INT(x) + 1.

enVision™ Algebra 1 | 26 | Selected Answers


PearsonRealize.com

Selected Answers
Topic 5

25. The graph of g is a vertical stretch Topic Review


​​ 54 ​​ that
of the graph of f by a factor of __
1. Check students’ work. See Teacher’s
is then translated right 2 units and up
Edition for details. 3. ceiling function
7 units.
5. piecewise-defined function
y
7. yes; (5, 5)
8 9. y x
4 −4 −2 O 2 4
x −2
O 2 4
−4

27. f(x) = –0.5| x – 4 | – 5 −6


29. g(x) = ∣x + 2∣ − 2 31. g(x) = −3∣x∣ −8
4.9
33. f(x) = −​____
​  3.85  ​​ |x − 3.85| + 4.9;
D: 0 ≤ x ≤ 7.7 35. absolute value; −7;  omain: all real numbers;
D
−15; (−15, −7); −22; 0 37. Part A If Range: y ≤ 0
there are 2 of your opponent’s ships on 11. (0, 0); maximum 13. Domain: all
the same horizontal line, you can use real numbers; Range: y ≤ 3;
the vertical line halfway between them Vertex: (0, 3)
as the axis of symmetry for an absolute
{2x,     x < 0
−2x, x ≥ 0
value function that passes through both 15. ​f(x) = ​ ​ ​​​​
ships. Part B Answers may vary. Sample: ​ 17. y
y = ∣x − 60∣ + 10​; ​y = −∣x − 40∣ + 90​; 6
​y = 2∣x − 50∣​; The red ship at (10, 60) is 4
captured by ​y = −∣x − 40∣ + 90​because
​−∣10 − 40∣ + 90 = −∣−30∣ + 90 = 60​; 2
The red ship at (30, 40) is captured by x
​y = ∣x − 60∣ + 10​because O 2 4 6
​∣30 − 60∣ + 10 = ∣−30∣ + 10 = 40​; −2
The red ship at (60, 70) is captured by ⎧ __ 5 ​x + 5,  x < 0
19. f​ (x) = ⎨
​y = −∣x − 40∣ + 90​because ⎪
3
  ​​​
​ ​ 
​−∣60 − 40∣ + 90 = −∣20∣ + 90 = 70​; 3
⎩−​  4 ​x + 3, x ≤ 0
⎪ __
The red ship at (90, 40) is captured by
​y = ∣x − 60∣ + 10​because 21. 8 23. 6
​∣90 − 60∣ + 10 = ∣30∣ + 10 = 40​. 25. y ; f(x) = ⌈​​ x⌉​​ + 5
Part C Answers may vary. Sample: Yes;
8
​y = ∣x − 60∣ + 10​captures the red ship
at (30, 40) and the red ship at (90, 40), 4
but the red ship at (30, 40) is also x
captured by ​y = 2∣x − 50∣​and the red O 2 4 6
ship at (90, 40) is also captured by
​y = −∣x − 40∣ + 90​.
enVision™ Algebra 1 | 27 | Selected Answers
PearsonRealize.com

Selected Answers
Topic 5

27. y 31. (2, 0)


20 y
6
16
4

12
x
−2 O 2 4 6
8 

33. (3, 1)
4 y
6
x
0
0 2 4 6 8 4


⎧1, 0 < x ≤ 12   2
x
2, 12 < x ≤ 24
O 2 4 6


3, 24 < x ≤ 36
29. f(x) = ⎨
​​   
​ ​​​​ 
4, 36 < x ≤ 48
5, 48 < x ≤ 60 35. The graph of g is a reflection
⎩6, 60 < x ≤ 72 across the x-axis of the graph of f and
translated 2 units right and 1 unit
 his function is like a ceiling function
T down. 37. The graph of g is a vertical
because you have to round up to the ​​ 32 ​​
stretch of the graph of f by a factor of __
nearest full dozen eggs to get the and translated 1 unit right and 8 units
number of cartons needed. down. 39. g(x) = 3∣x − 4∣ + 3
41. f(x) = ∣x – 5∣ + 4
43. y = −3∣x – 6∣ + 18; The domain is
0 ≤ x ≤ 12 because the graph is in the
first quadrant.

enVision™ Algebra 1 | 28 | Selected Answers


PearsonRealize.com

Selected Answers
Topic 6

Lesson 6-1 Lesson 6-2


1. The properties of rational exponents 1. An exponential function of the form ​
are the Power of a Power, Power of f(x) = ​ab​​  x​grows at an increasing rate
a Product, Product of Powers, and when ​b > 1​and a decreasing rate when
Quotient of Powers Properties. These ​0 < b < 1​. Exponential functions have
properties can be used to simplify an asymptote with equation ​y = 0​if the
expressions and solve problems range is ​y > 0​. 3. Answers may vary.
involving a power raised to a power, Sample: If ​b = 1​, then ​​b​​  x​is always 1,
a product raised to a power, or a and the function is a horizontal line.
product or quotient of the same base
5. y
raised to powers. 3. ​x = y​  5. When 8
simplifying expressions, it is easier if
6
the expression is written with rational
exponents instead of radicals so that 4
the properties of rational exponents
_1 _4 2
can be applied. 7. ​​7​​  ​  2 ​​​  9. ​​6​​  ​  3 ​​​  11. 2
x
13. ​x ______ = 6​  15. ​x = 2​  17. ​x = −3​ 
_1 –2 O 2 4 6
19. ​​ 2,500 ​​, ​​2,500​​  ​  3 ​​​  21. The student did

3

x
not distribute 3 in (​​2​​  3​​​)​​  x​ ​​  + 3​​ across ​ 7. ​f(x) = 4​​(__ ​  12 ​)​​​  ​ 9. 4 11. The student
x + 3​. The correct solution is ​x = −14​. incorrectly multiplied 6 and __ ​​ 13 ​​, so the
23. a. Answers may vary. Sample: exponent is applied to the product
​​a​​  0​​ ∙ ​​a​​  1​​ = ​​a​​  0 + 1​​ = ​​a​​  1​​ = a. Since
instead of just ​​ __13 ​​. 13. No; Answers may
​​a​​  0​​ ∙ a = a, the value of ​​a​​  0​​is 1 by the
Identity Property of Multiplication. vary. Sample: If the base b is greater
b. If you are adding two rational than 1, then as x increases, ​​b​​  x​​ increases.
exponents when applying the Product In this case, the function increases as
of Powers property and the sum is 0, x increases. If, however, b is less than
the value of the power is 1, because ​​ 1, then ​​b​​  x​​ decreases as x increases.
_1 _2 _2 In this case, the function decreases
a​​  0​​ = 1 25. ​​3​​  ​  2 ​​​ 27. ​​3​​  ​  5 ​​​ 29. ​​a​​  ​  3 ​​​
as x increases. 15. y-intercept: 1,
31. x = ___ ​​  60
7
 ​​ 33. no solution 35. x = –3 domain: all real numbers, range:
37. x = –4 39. x = 1 41. x = 2 ​y > 0​, asymptote: ​y = 0​; The function
43. Power of a Power Property increases as x increases.
45. approximately 4.54 in.
17. y
47. ​​(​x​​  ​  2 ​)​ ​ (​ ​72​​  ​  2 ​)​ ​ = 110​; Use Power of a
_1 _1
4
Product Property to simplify equation 2
to ​72x = ​110​​  2​​; ​x = 168.06​; x
_1
​​x​ ​2 ​​​ ≅ 12.96 cm 49. B −2 O 2 4
−2

enVision™ Algebra 1 | 29 | Selected Answers


PearsonRealize.com

Selected Answers
Topic 6

19. y growth and decay functions are both


200
modeled with ​f(x) = ​ab​​  x​. 13. Enter the
150 function on the Y= screen, display the
corresponding table, and find the value
100
of x where ​y = 15​. 15. ​f(x) = 20(1.25​)​​  x​​ 
50 17. After 20 years, the investment
x compounded quarterly will be worth
–4 –2 O 2 4 $668.22 more than the investment
21. f(x) = 2(4​​)​​  x​​ 23. ​f(x) = 5(2​)​​  x​​ compounded annually.
25. exponential; The function decreases 19. ​f(x) = 100​(0.95)​​  x​; x is about 10
by a constant ratio of __ ​​ 16 ​​. when ​f(x) = 60​  21. ​f(x) = 50​(0.9)​​  x​;
27. ​f(x) = 7​(0.9)​​  x​; At 10 weeks, the the average rate of change over
show will have about 2.44 million ​1 ≤ x ≤ 4​is about −4.065 and the
viewers, so it will be cancelled. average rate of change over
29. 3, ​y = 0​, all real numbers, ​y > 0​  ​5 ≤ x ≤ 8​is about −2.66. The rate of
31. Part A Buy new bulbs: change decreases as x increases because
​f(x) = 50x + 6​; Divide existing bulbs: it is an exponential decay function.
​f(x) = 6​(2)​​  x​ Part B Buy new plants: 23. ​f(x) = 4​(0.5)​​  x​; decay factor: 0.5
506; Divide existing plants: 6,144 25. ​f(x) = 100​(0.95)​​  x​; ​g(x) = 20​(1.05)​​  x​;
Part C Answers may vary. Sample: If The functions are equal at about ​x = 16​.
the gardener buys new bulbs, there will 27. ​f(x) = 500​(1.07)​​  x​; about 45 years;
be 256 plants in 5 years and 756 plants Enter the function in the Y = screen
in 15 years. If the gardener divides of a graphing calculator, press 2ND
the bulbs, there will be 192 plants in TABLE, and find the value of x where
5 years and 196,608 plants in 15 years. ​y = 10,000​. 29. ​f(x) = 240​(0.98)​​  x​;
The number of plants increases very ​f(x) = 180​(1.03)​​  x​
rapidly when existing bulbs are divided, y
so the gardener may want to revisit the 240
original strategy depending on how
200
many plants are wanted.
160
Lesson 6-3
120
1. savings accounts and investments
80
with compounded interest, population
growth and decline, the spread of a 40
computer virus or a viral video x
3. LaTanya confused growth factor O 2 4 6 8 10
with growth rate. The growth factor is
1.25. The growth rate is 25%. The number of students in both schools
5. ​f(x) = 100(1.05​)​​  x​ 7. ​f(x) = 512(0.5​)​​  x​​  will be about the same in about
9. $29.45 11. ​y = 0​; Exponential 6 years. You can find the approximate

enVision™ Algebra 1 | 30 | Selected Answers


PearsonRealize.com

Selected Answers
Topic 6
n−1
value of x where the graphs of the ​​ ​  n​​ = 3,000,000​​(__
43. a ​  13 ​)​​​  ​​; 9 hours
functions intersect. 31. D 45. D

Lesson 6-4 Lesson 6-5


1. Geometric sequences are exponential 1. When a constant is added to a
functions with a domain of the natural function, the graph is translated
numbers 1, 2, 3, … instead of the real vertically. The graph is translated up if
numbers. 3. Jamie reversed ​​a​  1​​​ and r in the constant is positive and down if the
the formula. It should be ​​a​  n​​ = 3​​(1.25)​​​  n−1​​. constant is negative. When a constant
5. geometric; the common ratio is 0.2. is subtracted from the exponent,
​  14 ​(​a​  n−1​​)​. The initial condition is ​​
7. ​​a​  n​​ = __ the graph is translated horizontally.
a​  1​​ = 640​. 9. ​​a​  n​​ = 3(​a​  n−1​​)​. The initial The graph is translated right if the
condition is ​​a​  1​​ = 1.25​. 11. Write the constant is positive and left if the
explicit formula ​​a​  n​​ = ​3​​  n−1​​or the constant is negative. 3. The graph
recursive formula ​​a​  n​​ = 3(​a​  n−1​​)​, ​​a​  1​​ = 1​. should be translated up 6 units. 5. By
13. The student forgot to include writing one of the general forms with
the initial condition ​​a​  1​​ = 210​in the addition and one with subtraction, the
recursive formula. 15. a. The n is translation is always in the positive
conventionally used to indicate that direction (up for vertical and right for
the domain of the function is the set of horizontal) for positive values of k and
counting or natural numbers 1, 2, 3, … h, and in the negative direction (down
rather than the real numbers. b. While for vertical and left for horizontal) for
the domain is limited to the natural negative values of k and h.
numbers, the range can include any 7. The graph is translated right 1 unit.
numbers. The numbers in the sequence 9. The graph is translated left 1 unit.
are the range of the sequence. 11. The graph is translated left 1 unit.
17. yes; ​​a​  n​​ = 1.5(​a​  n−1​​)​, ​​a​  1​​ = 8​  19. yes; ​​ 13. The graph is translated up 1 unit.
15. You could identify the number of
1
a​  n​​ = 3(​a​  n−1​​)​, ​​a​  1​​ = __
​  27  ​​  21. no 23. no units the y-intercept of g changes from
​  15 ​​ 
25. no 27. ​​a​  n​​ = 10(​a​  n−1​​)​, ​​a​  1​​ = __ the y-intercept of f, or you could identify
the number of units the asymptote of
​  23 ​​ 
​​ ​  n​​ = ​​5(​a​  n−1​​)​, ​​a​  1​​ = __
29. a
g changes from the asymptote of f.
n−1
​ ​  n​​ = ​100​(__
31. a ​  45 ​)​​​  ​ 17. a. The graph of g is a translation up
n−1 n−1 4 units and left 3 units from the graph of
​​ ​  n​​​ =
33. a 10​​​(__
​  59 ​)​​​  ​​  35. ​f(n) = 20​​(__
​  34 ​)​​​  ​​  f. b. The asymptote is affected by only a
n–1 vertical translation, so it becomes ​y = k​.
37. ​f(n) = 4​(2)​​  n−1​​  39. ​f(n) = 9​( ​  13 ​)​​​  ​
​ __
c. The domain of the function is the same
41. ​​a​  n​​ = 20(4​)​​  n−1​; ​a​  n​​ = 4(​a​  n−1​​), ​ as the parent function, and the range
a​  1​​ = 20;​Substitute 8 for n in of the function changes based on the
​​a​  n​​ = 20(4​)​​  n−1​: 20(4​)​​  8–1​ = 327,680​; There vertical translation. 19. translation 6 units
will be 327,680 shares after 8 hours. down 21. translation 1 unit right 23. 2

enVision™ Algebra 1 | 31 | Selected Answers


PearsonRealize.com

Selected Answers
Topic 6

25. y
the original by a factor of 2. You can
look at the new graph to estimate that
2
the value of x at ​y = 7,500​is about ​
f x
O
x = 30​. So, it will take about 30 years to
−4 −2 2
reach $7,500 with an initial investment
−2
g of $1,000.
−4
Topic Review
27. The graph of g is a translation up
3 units from the graph of f: 1. Check students’ work. See Teacher’s
y-intercept: 1; asymptote: ​y = 0​; Edition for details. 3. constant ratio
_
​  21 ​
5. compound ___36
interest 7. ​​8​​  ​​  9. ​​  5 ​​
range: y > 0; g: y-intercept: 4; ___
asymptote: ​y = 3​; range: ​y > 3​. 11. ​​ √64 ​​, 4
3

29. If h is positive, the graph is a 13. y


horizontal translation right by h
8
units of the graph of f(x) = ​​2​​  x​​. If h is
negative, the graph is a horizontal 6
translation left by |h| units of the graph
4
of f(x) = ​​2​​  x​​. 31. Enter g in a graphing
calculator and examine the table. Note 2
that g(x − 2) = j(x). Therefore, the x
graph of j is a translation 2 units right −2 −1 O 1 2
of the graph of g. j(x) = ​​2​​  x−1.5​​.
15. ​y = 0.5​(2)​​  x​​ 17. 6
33. a. y
19. f(x) = 200​​(0.85)​​  x​​ 21. $29,065.89,
j 8
f Annual: $28,965.96 23. no
6 ​​ ​  n​​​ = __
25. a ​​  15 ​​​​(2)​​  n−1​​
4 ​​a​  n​​​ = 2(​​a​  n−1​​​), ​​a​  1​​​ = __
​​  15 ​​
g
2 ​​ ​  n​​​ = 3.5(​​a​  n−1​​​), ​​a​  1​​​ = 6
27. a
x
29. translated down 5 units
–4 –2 O 2 4
31. translated right 2 units
b. y-intercepts: g: 0, j: 2 33. y
35. Part A The graph will be 8 g
translated 5 units to the right. Part B
6
15 years; Answers may vary. Sample:
f
If he invests now, it will take 10 years 4
to reach approximately $1,000. If the 2
graph shifts right 5 units, this same x
amount will be at ​x = 15​. Part C The O 2 4 6 8
new graph will be a vertical stretch of
enVision™ Algebra 1 | 32 | Selected Answers
PearsonRealize.com

Selected Answers
Topic 7

Lesson 7-1 45. Part A 18x + 8 Part B 10 ft, 9 ft,


14 ft, 7 ft, 24 ft, 16 ft
1. Adding and subtracting polynomials
is similar to adding and subtracting Lesson 7-2
rational numbers, except that you
must be careful to only add terms with 1. When multiplying polynomials, each
variables that have the same degree. term of one polynomial is multiplied
3. The standard form of a polynomial by each term of the second polynomial.
arranges the variables in order from This is similar to multiplying each
the term with highest degree to the place value of one number by each
term with lowest degree. The degree of place value of the other number.
the terms (monomials) determines the When adding the terms from the
order. 5. Answers may vary. Sample: multiplication of polynomials, only
To be like terms, terms must have the like terms can be combined, which
same degree AND the same variable. sometimes results in products with
7. cubic trinomial 9. ​​x​​  3​ + ​3x​​  2​ – 2x + 6​ two or more terms. However, when
11. ​​7x​​  2​ – 3x – 7​  13. The terms with multiplying rational numbers, all the
exponents of 2 would become zero, numbers are added, resulting in an
so the trinomial would become a answer with one term—a constant.
binomial. Since the square terms 3. The error is that Mercedes also
are zero, only exponents of degree multiplied the exponents. When
one (linear) remain (e.g., a quadratic multiplying terms with variables,
trinomial plus a quadratic trinomial can the exponents are added, so the
be a linear binomial): answer should be: ​4​x​​  6​ + 8​x​​  5​ − 12​x​​  3​​.
​​(3x​​  2​ + 2x + 9) + (−​3x​​  2​ − 5x − 8) = 5. ​–​6x​​  5​ + ​8x​​  4​ – ​16x​​  3​​  7. ​​3x​​  2​ – 2x – 8​ 
−3x + 1​  15. The student did not 9. ​​6x​​  3​ –​5x​​  2​ – 16x + 15​ 
distribute the (​−​1) to all terms in the 11. (x + 7)(x + 4) 13. The student
second polynomial. They only distributed multiplied the first terms of each
it to the first term. The second line binomial and the second terms of each
should read ​−5​x​​  2​ + 2x − 3 − 3​x​​ 2​ + 2x + 6​. binomial together instead of distributing
The answer should be​−8​x​​ 2​ + 4x + 3​. each term of the first binomial to each
17. a. never b. sometimes c. never term of the second binomial.
d. always 19. 1 21. 0 23. cubic trinomial ​2x(4x) + 2x(−1) + 2(4x) + 2(−1)​ 
25. quadratic binomial 15. No; The product of a monomial
27. ​​5x​​  2​ – 6x​ 29. –2z 31. 10b – 4 and trinomial can never be a binomial.
33. ​4​y​​  2​– 3y – 2​ 35. ​​6m​​  2​– 1​ The product will be a trinomial.
37. ​​6x​​  2​ + 144x + 864​ 39. 6x – 20
41. a. sum and difference: degree 4,
trinomial b. sum and difference: degree
4; The number of terms in each would
depend on the degrees of the second
terms in Polynomials A and B. 43. C
enVision™ Algebra 1 | 33 | Selected Answers
PearsonRealize.com

Selected Answers
Topic 7

17. Answers may vary. Sample: Find 3. The terms of the two binomials
the volume of each rectangular prism are the same so that when they are
by first multiplying the binomials. Then multiplied the products are perfect
multiply the resulting trinomials by the squares. The middle terms with x have
monomial side. Finally, combine like opposite signs and cancel each other,
terms to find the combined volume of leaving two perfect squares. The
the two prisms. 19. ​​3y​​  3​ – ​2y​​  2​ + 7y​ second term of the product is always
21. ​​–10x​​  6​ + ​20x​​  5​ –​  10x​​  3​ negative. So, the final product is a
difference of two perfect squares.
23.
4x +1 5. ​​x​​  2​ −14x + 49​ 7. ​​x​​  2​ −16​
2x 8x2 2x 9. ​2,916 ​cm​​  2​​ 
+1 4x +1 11. ​(x + 9)(x + 9) = ​x​​  2​ + 36x + 81;
​​(x − 7)(x − 7) = ​x​​  2​ − 14x + 49​;
The product is ​8​x​​  2​ + 6x + 1​. ​(2x − 1​)​​  2​ = 4​x​​  2​− 4x + 1​  a. All are a
25. ​​6x​​  2​ + 7x – 20​ trinomial; the first and third terms
27. ​​2y​​  3​ + ​3y​​  2​ – 5y +12​ are perfect squares. b. Yes; because
29. ​​–6x​​  4​ + ​17x​​  3​ – ​16x​​  2​ + 6x​ the factors are the same, either two
31. ​​3x​​  4​​ + ​​7x​​  3​​ – ​​2x​​  2​​ + 12x negatives or two positives are being
33. ​​18x​​  2​ + 48x – 50​  35. Answers multiplied, resulting in a positive
may vary. Sample: Change ​​x​​  2​ + 2x − 4 answer. c. The sign of the second
to ​x​​  2​ + 4x − 4​. The new product is: term in the product is the same as the
(​2x + 2​)(​​x​​  2​ + 4x − 4​) sign of the binomial being squared.
​= 2​x​​  3​ + 8​x​​  2​ − 8x + 2​x​​  2​ + 8x − 8 d. All the exponents are even. 13. The
= 2​x​​  3​ + 10​x​​  2​ − 8​. student only multiplied the first terms
37. ​​10x​​  2​​ + 16x – 24 39. B together and the last terms together.
Lesson 7-3 He did not multiply the first and last
terms together. The correct answer is ​​
1. For the product of the square of a x​​  2​ + 10x + 25​. 15. a. Yes; consecutive
binomial, the middle term is always even numbers are 1 away from an odd
equal to twice the product of a and number, which is the average between
b, while for the product of a sum and them. You can write their product as
a difference, the middle terms always the sum and difference of their average
cancel, leaving only two terms in the and 1. For example; ​6 • 8 = (7 − 1)
product of the expression, ​​a​​  2​​ and ​​b​​  2​​. (7 + 1) = ​7​​  2​ − 1 = 49 − 1​. So 48 is one
less than the perfect square 49. b. Yes.
In a similar argument, consecutive odd
numbers are each 1 away from an even
number. For example, 7 ​• ​9 = (8 + 1)
(8 ​−​1) = ​​8​​  2​ − 1​. So 63 is one less than
the perfect square 64. 17. ​25​x​​  2​ − 30x + 9​

enVision™ Algebra 1 | 34 | Selected Answers


PearsonRealize.com

Selected Answers
Topic 7

19. ​​x​​  2​– 26x + 169​ 21. ​​9k​​  2​ + 48k + 64​ 33. 6​​x​​  2​​​​y​​  2​​(4x − 5y + 2​​y​​  2​​) 35. 6x and
23. ​4​a​​  2​ +12ab + 9​b​​  2​​ 2
x − 3 37. 36 yd 39. 10​ ​​ x​​  ​ − 4x​​​ 
25. ​0.16​x​​  2​ + 0.96x + 1.44​  27. 5,184 2x(5x − 2)
29. ​4​x​​  2​– 25​ 31. ​​x​​  4​– 4​y​​  2​ 2
41. 3​​x​​  ​​; 9; 3; 6; 2; 12; 2; 6; 2
33. ​​x​​  2​– 6.25​ 35. 8,051 43. Part A Face A: 3x + 1 by x;
37. (8x + 16) square units 39. a. x by Face B: x + 1 by x; Face C: 3x + 1 by x + 1
​(x − y)​ and y by ​(x − y)​  b. ​​x​​  2​− xy​ and Part B 3​​x​​  2​​ + 4x + 1
​xy − ​y​​  2​​  c. The remaining figure is the
Part C 14​​x​​  2​​ + 12x + 2
result of finding the difference between
the larger ​​x​​  2​​and the smaller ​​y​​  2​​. Part D 3​​x​​  3​​ + 4​​x​​  2​​ + x
41. yes; no; no; yes; no
Lesson 7-5
43. Part A Answers may vary.
Sample: the answers are odd 1. You can use patterns to determine
numbers. Part B 22, 23 Part C The whether factors are positive or
answers are even numbers that increase negative, giving you a starting
by 8. Part D 24, 26 point. 3. Factor out any common
factors. Factoring out any common
Lesson 7-4 factors allows you to more clearly
1. In both instances, the product of the see what patterns to use when
common factors between either the factoring. 5. 1 and 16, 2 and 8,
integers or polynomials become the 4 and 4 7. both negative
GCF. 3. 1 5. No, while 2 is the GCF of 9.
Factors of 12 Sum of Factors
6 and 8, it is not the GCF of ​​x​​  6​​ and ​​x​​  8​​; ​​
−1 and −12 −13
x​​  6​​represents 6 factors of x and ​​x​​  8​​
−3 and −4 −7
represents 8 factors of x, so they have 6
factors of x in common, or ​​x​​  6​​. 7. ​​x​​  3​​y −2 and −6 −8

9. 1 11. ​​x​​  6​​​​y​​  8​​  13. −3​​x​​  2​​(​​x​​  2​​ − 4x + 7) 11. The binomial factors correspond to
15. ​​x​​  8​​(​​x​​  2​​ + x − 1) 17. 50​​a​​  7​​​​b​​  3​​(2​​b​​  2​​ − 3a) the length and width of the rectangle
that can be formed by the algebra
19. Answers may vary. Sample:
tiles that represent the trinomial.
​12​x​​  4​ + 32​x​​  3​ − 24​x​​  2​​  13. The student found the factors of b
21. ​2x(7​x​​  3​ − 10x + 5)​ 23. (x + 5) rather than the factors of c. The correct
25. 1: {2x, 3x, 4x, 5xy, 7x, 9y, 12xy, factored form of ​​x​​  2​ − 11x − 26​ is
13x, 15x}; 2x: {2x, 4x, 12xy}; ​(x − 13) (x + 2)​. 15. Sample: multiply 4
by ​​y​​  2​​. 17. (​x + 3​) inches by (​x + 4​)
3: {3x, 9y, 12xy, 15x}; 4x: {4x, 12xy};
inches; Answers may vary. Sample:
5x: {5xy, 15x}; y: {5xy, 9y} Because the area of a rectangle is the
27. 3​​ab​​  2​​ 29. ​​x​​  2​​​​y​​  3​​ 31. −2y(2​​y​​  3​​ − 3y + 7) product of its length and its width,
factoring the total area into two
factors gives you possible dimensions of
the rectangle. 19. (x + 1)(x + 1)

enVision™ Algebra 1 | 35 | Selected Answers


PearsonRealize.com

Selected Answers
Topic 7

21. Lesson 7-6


Factors Sum of Factors
–1 × 22 21 1. In both situations, you are finding
1 × –22 –21 factors of ac that sum to b and using
–2 × 11 9
those to factor the trinomial. However,
when ​a = 1​, those factors correspond
2 × –11 –9
to the constant terms in the binomial
23. (x − 8)(x − 3) 25. (x − 10)(x − 3) factors. When a does not equal 1, you
27. (x + 2)(x − 4) 29. (x − 9)(x − 3) must first rewrite b and then factor out
31. (x − 14y)(x − 2y) 33. (x + 12)(x + 4) common factors to arrive at the two
35. ​(x + 9)(x + 6y)​  37. a. (x + 8) feet by binomial factors of the trinomial.
(x − 5) feet b. ​3x − 2​; Add (​x + 8​), 3. Both binomial factors have a
(​x − 5​), and (​x − 5​) to find the total constant term of 1. 5. Since the entire
length of rope. No rope is needed for expression can be rewritten in terms of
the side along the beach. c. 80 ft addition, and addition is commutative,
the order of the terms does not matter.
39. I. x2 + 13x + 30 A. (x − 10)(x + 3) The factored form will be the same
II. x2 + x − 30 B. (x − 6)(x + 5) whether Felipe puts 7x or −8x first.
7. −1 and 24, 1 and −24, −2 and 12,
III. x2 − 7x − 30 C. (x − 5)(x + 6) 2 and −12, −3 and 8, 3 and −8, −4 and
IV. x2 − x − 30 D. (x + 10)(x + 3) 6, 4 and −6 9. no 11. ​18x + 2x​  13. 3x
+ 5 and 2x − 1 15. −22, −10, 22 and 10
41. Part A field: (​x + 5​) by (​x + 90​); 17. The student used the method for
picnic area: (​x + 15​) by (​x + 30​); factoring ​a​x​​  2​ + bx + c​when ​a = 1​even
playground: (​x + 15​) by (​x + 20​) though a is not equal to 1. The correct
Part B Find the sum of the areas that factored form of ​2​x​​  2​ + 11x + 15​is
are given or use the dimensions you ​(2x + 5)(x + 3)​.
found to find the dimension of the
park and then multiply to find the total 19.
x2 x2 x x x x2 x2 x x x
area. Part C (x + 90) ft by (2x + 20) ft
Part D Yes; Sample: You know that x x 1 1 1
x2 x2 x x x
x + 90 is the same length as 2x + 50.
Set these expressions equal to one x2 x2 x x x x x 1 1 1
x x 1 1 1
another and then solve for x. x x 1 1 1

21. (​ x + 1)(2x + 5)​ 23. ​4(x + 1)(x + 3)​ 


25. 3 ​ (x + 7)(x – 4)​  27. 2 and 7

enVision™ Algebra 1 | 36 | Selected Answers


PearsonRealize.com

Selected Answers
Topic 7

29. ​−​4 and 2 31. 5 and ­–21 19. ​(2x − y)(2x + y)(​4x​​  2​ + ​y​​  2​)​;
33. ​(4x +1)(x + 3)​ 35. ​(2x − 1)(x + 4)​ difference of two squares followed by
37. ​3x(2x + 1)(x + 1)​ difference of two squares.
39. ​(6x + 5)(2x + 1)​ 41. 7​(3x + 1)(x – 2)​
21.
43. ​(9x + 1)(x + 5)​ 45. ​(3x − 2y)(x + y)​
47. (5x + y)(x – y) 49. (​2x − 3​) by (​x + 8​); a−b

(​2x − 1​) by (​x + 10​); ​2​x​​  2​ + 19x −10 ​ft​​ 2​​  b a−b


51. a. (​2x + 8​) in. by (​2x + 10​) in.
a
b. 8 in. by 10 in. c. The photographer b a b

may not know how wide she wants a−b


a+b
the frames of the photos to be, or she a
may vary the width of the frame for
23. 3x + 4; Since the width is twice
different photos. 53. B
the length, the rectangle is made of
Lesson 7-7 two squares side-by-side. If you divide
the area by 2, the resulting expression
1. In a perfect square trinomial, both
will be the area of each square,
the first and last term must be perfect
​​9x​​  2​ + 24x + 16​, which is a perfect-square
squares and the middle term must be
trinomial. Factor the perfect-square
twice the product of the square root
trinomial to find the length of each
of the first term and the square root
square, which is also the length of the
of the last term. In the difference of
rectangle. 25. 144 27. 54 29. ​(6x + 10)​
two squares, both the first term and
31. ​(x + 8​)​​  2​​ 33. ​(x − 9​)​​  2​​
the second term in the binomial must
35. (10x + 6)(10x − 6) or 4(5x + 3)(5x − 3)
be perfect squares. 3. Answers may
37. 8​(x − 2​)​​  2​​ 39. 2x​(x + 8​)​​  2​​
vary. Sample: Both are the square of
41. x(7x + 4y)(7x − 4y) 43. ​−3x(x – 3​)​​  2​
an expression. No, for (ax + b​​)​​  2​​, where
both a and b are not equal to zero, the 45. ( ​ 12 ​)​​(x + __
​​ x − __ ​  12 ​)​​
expanded expression will always be a 47. ​​(p − ___
​ 10 )(
7  ​ ​​ p + ___
)
7  ​ ​​
​  10
trinomial. 5. Factoring out the greatest
49. a. ​(x + 16)​by ​(x + 16)​; yes; The
common factor results in a polynomial
factors are equal so the side lengths are
with smaller coefficients and/or
the same. b. ​(x + 2y)​ by ​(x − 2y)​; yes,
smaller exponents of the variable(s).
but only when ​y = 0​; The factors are
This makes it easier to analyze the
equal when ​y = 0​. Other than that, they
polynomial or factor it further.
are not equal. c. ​(x − 10)​by ​(x − 10)​;
7. perfect-square trinomial 9. perfect-
yes; The factors are equal so the side
square trinomial 11. perfect-square
lengths are the same. 51. a. 100​​p​​  2​​− ​​n​​  2​​ 
trinomial 13. ​4(3x + 2​)​​  2​​ 
b. 10p, 10p, 10p − n, n, n, 10p − n
15. ​2(6x + 4)(6x − 4)​  17. Answers may
vary. Sample: You can use a difference
of two squares to rewrite
​​50​​ 2​​ − ​​45​​ 2​​ as ​(50 − 45)(50 + 45) = 5(95)​.

enVision™ Algebra 1 | 37 | Selected Answers


PearsonRealize.com

Selected Answers
Topic 7

c. ​(10p + n)​by ​(10p − n)​; Because the 39. Sample answer:


area of the office is ​100​p​​  2​ − ​n​​  2​​,
Factors Sum of Factors
you can use the difference of two
squares pattern to find the dimensions −1 and 18 −17
of a rectangular office with the same 1 and −18 17
area. 53. C −2 and 9 7

Topic Review 41. (x + 7)(x – 4) 43. (x + 3y)(x + 15y)


1. Check students’ work. See Teacher’s 45.
Factors Sum of Factors
Edition for details. 3. perfect-square
trinomial 5. difference of two squares 20 and 1 21
7. linear 9. cubic 11. ​​–x​​  2​ – 12x + 3​ 10 and 2 12
13. 5x; Answers may vary. Sample: In 5 and 4 9
the sum, the coefficient of the x term is
47. (3x + 4)(x + 2) 49. (5x – 3)(x + 2)
8. So far, there is only 3x represented in
51. (5x + 4)(2x – 1) 53. 23, –23, 10,
the addends, so missing term is 5x.
–10, 5, –5, 2, –2 55. 64 57. ​​(x + 5)​​  2​​ 
15. ​​x​​  2​ + 2x – 35​  17. ​​20x​​  2​ – 11xy – ​3y​​  2​​ 
59. ​(x – 9)​​  2​​  61. ​3​(x + 3)​​  2​​  63. No, 3
19. x; 8 21. ​​b​​  2​ + 24b + 144​ 
is not a perfect square so it is not a
23. ​​36x​​  2​ – 81​  25. ​​2.25x​​  2​ – 4​  27. 25
difference of two squares. Also, ​​3x​​  2​​
29. 3x 31. ​​7xy​​  2​​  33. ​3x(​5x​​  2​ – 14)​ 
and 49 share no common factor.
35. ​6a(​2a​​  2​ + 3a – 6)​  37. Answers may
vary. Sample: ​​9x​​  6​ – ​27x​​  3​ + 6x​

enVision™ Algebra 1 | 38 | Selected Answers


PearsonRealize.com

Selected Answers
Topic 8

Lesson 8-1 29. B, C 31. Part A Design A:


​f(x) = 0.03​x​​  2​; Design B: ​g(x) = 0.024​x​​  2​​;
1. ​f(x) = ​x​​  2​​; a parabola that opens
in each function, the value of a is the
upward, with axis of symmetry ​x = 0​,
product of the cost per square inch
minimum, vertex, and x- and
and 6, because ​​x​​  2​​models the area of a
y-intercepts at (0, 0). The function
square with side length x, and a cube
decreases over the interval ​x < 0​and
has 6 square sides. Part B The average
increases over the interval ​x > 0​.
rate of change is 0.42 for Design A and
3. The word “parent” is included
0.336 for Design B. This means that for
because all other quadratic functions
every increase of 1 inch in side length
are related to the parent function.
between 6 and 8 inches, the cost of the
5. The graph is narrower than the
box increases by $0.42 using Design
graph of the parent function. 7. The
A and by $0.336 using Design B. The
graph is narrower than the graph
average rate of change for Design A
of the parent function and opens
is 1.25 times as great as the average
downward. 9. 10; the graph of the
rate of change for Design B. Part C
function opens upward. 11. The
Answers may vary. Sample: Regardless
coordinates used to calculate the
of the side length, the packaging cost
slope are incorrect. Use (–4, 8) and
for Design A is 1.25 times as great as
​(–2, 2)​. The average rate of change
the packaging cost for Design B.
is –3. 13. a. sometimes true b. always
true c. always true 15. The graph Lesson 8-2
is wider. 17. The graph is wider and
1. You can use the values of h and k
opens downward. 19. The graph
from the vertex form to determine the
is narrower. 21. decreasing: ​x < 0​;
locations of the vertex and the axis
increasing ​x > 0​  23. ​A(x) = 0.5​x​​  2​​;
of symmetry. You can use the value
3.125 units2 25. The average rate of
of a to determine whether the graph
change of g, 24, is 2 times greater than
opens upward or downward, and use
the average rate of change of f, 12.
the value of a to locate points on the
27. a. 24.36; 34.8 b. Between
graph 1 unit to the left of and 1 unit
diameters of 6 and 8 inches, the
to right of the vertex. 3. Answers
number of calories in a tortilla
may vary. Sample: The vertex and line
increased by an average of 24.36
of symmetry are the same for both
calories per inch of diameter. Between
functions. The quadratic function is
diameters of 9 and 11 inches, the
curved and continuous, while the
number of calories in a tortilla
absolute value function is not.
increased by an average of 34.8
calories per inch of diameter. c. As
the diameter of a tortilla increases,
the number of calories in the tortilla
increases more and more rapidly.

enVision™ Algebra 1 | 39 | Selected Answers


PearsonRealize.com

Selected Answers
Topic 8

5. y 35. y
16
6
12
4
8
2
4 x
x −4 −2 O 2
−4 −2 O 2 4

7. y 37. f(x) = 2(x + 1​​)​​  2​​ – 4 39. (–1, 42) 41. If


2 the player is 2 ft away from the net, the
x net is located at x = 1, and the height of
−4 −2 O the ball is f(1) = 7.75, or 7 ft, 8 in., so the
−2 ball will go over the net. If the player
hits the ball from 4 feet away, the net is
−4
located at x = 3, and the height of the
ball is f(3) = 7.75, or 7 ft, 8 in., so the
ball will go over the net. 43. B
9. ​f(x) = –16(x – 1​)​​  2​ + 22​  11. No;
because |a| > 1, the graph of this Lesson 8-3
parabola is narrower than the graph of 1. The standard form of a function is
the parent function f(x) = ​​x​​  2​​. the expansion of the vertex form.
13. f(x) = 2(x – 1​​)​​  2​​ – 3 15. (0, 2); x = 0 3. Square the binomial in the vertex
17. (0, –1); x = 0 19. (0, –2.25); x = 0 form, multiply using the Distributive
21. (0, 7); x = 0 23. (0, –2); x = –2 Property, and simplify.
25. (0.5, 0); x = 0.5 27. ​f(x) = ​(x – 2 )​​  2​ + 3​
29. (–1, 4); x = –1; opens upward; 5. y
8
narrower than the width of the graph
of f(x) = ​​x​​  2​​  31. (5, 6); x = 5; opens 4
downward; wider than the width of the x
graph of f(x) = ​​x​​  2​​ −8 −4 O 4 8
−4
33. y
8
−8
6
axis of symmetry: x = –2; y-intercept: –1;
4
vertex: (–2, –9)
2
x
−2 O 2 4

enVision™ Algebra 1 | 40 | Selected Answers


PearsonRealize.com

Selected Answers
Topic 8

7. y 39. 8.5 ft; Sample answer: The


x lowest point above the ground is the
−4 −2 O 2 y-coordinate of the vertex. Find the
−2 –b
x-coordinate: ​​ __
2a
​  1  ​​ =​​ ___
 ​ = ______ 1
0.5
 ​ = 2​. Then
2(0.25)
−4 find the y-coordinate: f(2) = 8.5. 41. D
−6 Lesson 8-4
1. area and vertical motion problems
axis of symmetry: x = –1; y-intercept: –5; 3. The object is dropped.
vertex: (–1, –2) 5. ​h(t) = –16​t​​  2​ + 32t + 20;​36 ft; 1 s
9. No; the maximum height of the 7. A(x) = 4​x​​  2 ​+ 30x + 36
balloon is 9 ft. 11. standard form; c is 9. a. 650 ft; 6.25 s b. 24.25 ft; 1.125 s
the y-intercept. 13. (1, –2); The exact 11. In a graph of a vertical motion
axis of symmetry can be found to be model, the x-axis represents time, not
​  –2  ​​ = 0.8; Then, substitute
x = ​– ______ horizontal distance. 13. a. length: ​x + 5​,
2(1.25) width: ​x – 2​ b. Answers may vary.
0.8 into f(x) ​= 1.25​x​​  2​​ – 2x – 1;
Sample: Since the width, ​x – 2​, must be
​f(0.8) = 1.25(0.8​)​​  2​​ – 2(0.8) – 1 = –1.8; positive, a reasonable domain is ​x > 2​,
(0.8, –1.8) 15. −6 17. −7 19. 3 and a reasonable range is ​f(x) > 0​.
21. 2; x = −2; (−2, −6) 23. 0; x = −2;
(−2, −1.6) 25. 12; x = −0.5; (−0.5, 10.75) 15. ​f(x) = 3​x​​  2​ – 3x – 18​
27. 12; x = 3; (3, 3) 29. g; minimum of g
y
is −5; minimum of f is −4. 6
31. g; maximum of g is 5; maximum of x
f is 4. 33. ​f(x) = 4​x​​  2​ + 8x + 1​ −4 −2 O 2 4
35. ​f(x)​= –2​​x​​  2​​ + 36x – 147 37. Ball B; −6
1 m; Answers may vary. Sample: The −12
maximum height is the y-coordinate
–b
of the vertex. Substitute __ ​​ 2a ​​ into −18
–b
function f: __
​​  2a ​  –14.7  ​ = 1.5, so 1.5 is
 ​ ​= ______
2(–4.9)
f(8) = 150
the x-coordinate of the vertex, and
f(1.5) = 12, so the maximum height for 17. h(t) = –16​​t​​  2​​ + 200t; 625 ft
Ball A is 12 m. The table shows that 19. h(t) = –16​​t​​  2​​ + 48t + 6; 42 ft
function g has symmetry about
​x = 1.5​, so (1.5, 13) is the vertex and the
maximum height for Ball B is 13 m.

enVision™ Algebra 1 | 41 | Selected Answers


PearsonRealize.com

Selected Answers
Topic 8

21. Part B Yes; The maximum height of the


x y f(x) Residual
ball is about 25 ft, so it will land on the
−2 −6 −8 +2 upper deck.
−1 −1 −2 +1
0 3 2 +1 Lesson 8-5
1 4 4 0 1. Look at the first differences,
2 3 4 −1 second differences, and the ratios
of consecutive terms. 3. Data with
y
4 constant first differences should be
modeled with a linear function, not
2
a quadratic function. 5. linear
x
7. A flow chart should show the following
−4 −2 O 4
reasoning. IF the first differences are
−2
constant, then the function is linear; ELSE
−4 IF the second differences are constant,
then the function is quadratic; ELSE IF the
The residuals are on or close to the common ratios are constant, then the
x-axis, which indicates the function may function is exponential; ELSE the function
be a good fit. is not linear, quadratic, or exponential.
23. y = .0.0001​​x​​  2​​ − 0.3791x + 603.8225 9. For analysis of first differences to
25. Answers may vary. Sample: No; a give good information, the differences
price of $15 is too high. Using quadratic of the related x-values must also be
regression on a graphing calculator constant. If the student only considers
gives the model the data for x = –3, –1, 1, and 3, then
​f(x) = –2.57​x​​  2​ + 67.57x – 182.8​. The price a linear, quadratic, or exponential
generating the highest profit is the function would not model the
x-value of the vertex, data. 11. exponential 13. quadratic
–b 15. linear; f(x) ​= 2.5x – 20.02​ 
which is __​​  2a  ​​, which is about 13.15. So,
the price that will maximize profits 17. a. f(6) = 4.5, f(8) = 6, f(12) = 9;
should be about $13. 27. 10, 96, 154, 3. g(6) = 3.375, g(8) = 6, g(12) = 13.5;
h(6) ≈ 3.815, h(8) ≈ 5.960, h(12) ≈ 14.552
29. Part A ​h(t) = –16​t​​  2​ + 35t + 6​ b. Starting at about x = 10.3, function h
h exceeds functions f and function g.
24
19. The budget will not be sufficient.
18 Using quadratic regression, the
function ​f(x) = 2.65​x​​  2​ + 3.1x – 40​
12
models the data with correlation
6 coefficient r = 0.999. Based on the
t model, the cost of the 150-meter-wide
−4 −2 O 2 4 parking lot is $60,050. 21. A

enVision™ Algebra 1 | 42 | Selected Answers


PearsonRealize.com

Selected Answers
Topic 8

23. Part A App A: ​f(x) = ​3​​  x​; 11. Both graphs open downward and
App B: ​g(x) = 998x + 50​; have vertex (3, 2). The graph of f is
App C: ​h(x) = 10​0x​​  2 ​+ 75x + 2,500​; narrower than the graph of g.
App A is exponential because ratios 13. (5, –2); x = 5 15. 5; x = 2; (2, 9)
of consecutive y-values are constant, 17. If a > 0, then the parabola has
App B is linear because first differences a minimum value. If a < 0, then the
are constant, and App C is quadratic parabola has a maximum value.
because second differences are 19. The ball was tossed at 5 ft.
constant. ​2​1​. ​h(x) = –16​x​​  2​+ 18x + 9; 14.1 ft
Part B App A will take the greatest ​2​3​. ​f(x) = ​2x​​  2​ – x – 3​
amount of time as the number of data
items to be analyzed increases; Answers y
will vary. Sample: The conjecture could 12
be supported using a graph of the 8
three functions.
4
Topic Review x
−4 −2 O 2 4
1. Check students’ work. See Teacher’s
−4
Edition for details. 3. quadratic
parent function 5. standard form of
a quadratic function 7. The graph of The vertex (0.25, –3.125) represents
coefficient. However, ​g(x) = 1.5​x​​  2​​ is the minimum area of the rectangle
narrower. 9. When a > 0, the function when coefficient. However, x = 0.25
has a minimum value. When a < 0, if the value of x = 0.25 is possible for
the function has a maximum value. the rectangle, which it is not. The
The maximum or minimum value is x-intercepts, –1 and 1.5, represent
always (0, 0). when the area of the rectangle is 0.
domain: x > 1.5; range: y > 0
25. Determine if the differences in
the x­-values are constant. 27. linear

enVision™ Algebra 1 | 43 | Selected Answers


PearsonRealize.com

Selected Answers
Topic 9

Lesson 9-1 Lesson 9-2


1. For graphs, the zeros of the related 1. If a quadratic equation can be
function are the solutions. For tables, factored, you can use the Zero-Product
the values of x when the function Property. The two linear factors can
value is 0 are the solutions. If exact be set to zero and solved to find
values don’t give a function value of the solution(s). 3. The Zero-Product
0, look where sign changes occur and Property says that for all real numbers
find approximations. 3. Eli found the a and b, if ab = 0, then either a = 0
solutions of ​​x​​  2​ − 100 = 0​. The equation or b = 0. If you can factor a quadratic
has no solution. 5. 1 7. ​−​1, 3 9. ​−​4, 1 equation that has 0 on one side,
11. approximately 2.2 13. There is no then each factor set equal to 0 is a
solution. solution. 5. 10, –20
15. Answers may vary. Sample: 7. (x + 16) (x + 2) = 0; –16, –2 9. –1
y ​​  52 ​​  , –3 13. (x + 1)(x – 7) = 0
11. __
2 15. x – 8 must be a factor;
x Substitute 8 into the quadratic
−2 O
equation and see if it makes the
equation true or factor the equation
−4 to see if x – 8 is a factor.
17. The student did not get 0 on one
17. 2 19. −3, 2 21. 2 23. 0, 5 25. 1, 7 side of the equation.
27. 1.5, 4 29. approximately −2.1,
approximately 2.1 31. −4 33. no x​​ ​​  2​​ + 2x – 3 = 5
solution 35. No, the solutions to ​​x​​  2​​ + 2x – 8 = 0
the equation 0.5​​n​​  2​​ + 0.5n = 50 are (x – 2)(x + 4) = 0
not integers, because the related x – 2 = 0 or x + 4 = 0
quadratic equation ​​n​​  2​​ + n − 100 = 0 is       x = 2 or x = –4
not factorable. A solution must be a
positive integer for it to be a triangular 19. a. (x – 2)(x – 6) = 0; the orange
number. curve, with vertex of (4, –2)
37. no; yes; yes; no; yes ​​  12 ​​  (x – 2)(x – 6) = 0 c. The graph of
b. __
39. Part A ​−16​x​​  2​ + 96x + 256 = 0​; the second equation is compressed by a
​x = −2, x = 8​  Part B No; −2 is a solution factor of __ ​​  12 ​​ , which is the constant factor.
of the equation, but not the problem, ​​  5 ​​  , –​​ __
21. __ 8 ​​ 25. −2, 7 27. __
2 ​​  23. ​​ __ ​​  1 ​​  , 2
because time cannot be negative. 2 7 3 2
1 ​​  31. (24 – 2x)(12 – 2x) = 189;
29. –3, –​​ __
Part C 400 ft; after 3 seconds 5
1.5 in.

enVision™ Algebra 1 | 44 | Selected Answers


PearsonRealize.com

Selected Answers
Topic 9

33. (x + 7)(x + 9) = 0; (–8, –1) 3. Write the prime factorization:


_____________
y ​​√2
  ∙ 2 ∙ 2 ∙ 2 ∙ 2 ​​
6 Remove each pair of 2s inside the
radicand and write one factor of
4
2 outside the radicand for each pair.
_____________ __
2
= 2 ​∙​2​​√  
2 ∙ 2 ∙ 2 ∙ 2 ∙ 2 ​​= 4​​√2
 ​​
x
−16 −12 −8 −4 O 5. No; Since 9 is a factor of 45 and 9
is a perfect square, the radical can be
__ __ ___
35. a. –(x – 3)(x – 8) = 0 b. (5.5, 6.25) rewritten as 3​​√5 ​​. 7. ​4√ ​ 5 ​​  9. ​2​x​​  2​ √
​ 10 ​​ 
__ __ __
c. Multiply by 16; 11. ​10​√2 ​​  13. ​12​x​​  2​ √ ​ 6x ​​  15. ​3​x​​  4​ √
​ 70 ​​ 
–16(x – 3)(x – 8) = 0. The equation has 17. The expressions are not equivalent;
the same roots, but the vertex is (5.5, ___ __ ____ ______
5​​√28 ​​ = 10​​√7 ​​ and √ ​​ 119 ​​ = √ ​​ 17 ∙ 7 ​​ 
16 ∙ 6.25), or (5.5, 100). __
19. 4​​x​​  4​​​​y​​  4√
​​​​ 2 ​​  21. In the next-to-last step,
37. I. B, D; II. A, D; III. B, E; IV. A, B; V. A, C
2 was incorrectly removed from the
39. Part A Check students’ equations. square root.
______________________
The starting point of the water is at
4​​√7
  ∙ 2 ∙ 7 ∙ x ∙ x ∙ x ∙ x ∙ x ​​
(1, 0) so the ending point is (6, 0). Both ___________
points must satisfy students’ equations, = 4 ∙ 7​​√2
   ∙ ​x​​  2​∙ ​x​​  2​∙ x ​​
___
and the leading coefficient must be = 28​​x​​  2√
​​​​ 2x ​​
negative. Part B The height of the 23. Radicals are equivalent to
water is the y-value from the vertex __
​​ x ​​ = ​​x​​  ​   ​​​. So multiplying
_
exponents: √
1
2

based on the equation from Part A. radical expressions is the same as


The distance from the edge of the pool multiplying exponential expressions.
to where the water hits the center is ____ _____
25. a. 2​​x​​  2​​ ​​ √3​x​​  2​ ​​; Factor ​​ √24​x​​  8​ ​​.
3 3
5 ft, or half the width of the pool. ______________________
Part C Check students’ work. = 3​​ √​2
  ​​  3​∙ 3 ∙ (​x​​  2​∙ ​x​​  2​∙ ​x​​  2​) ∙ ​x​​  2​ ​​
____
= 2​​x​​  2√ ​​​​ 3​x​​  2​ ​​
Lesson 9-3 ___ ______
b. 9​​x​​  3​​ 4​​ √3x ​​; Factor ​​√​3​​  9​​x​​  13​ ​​.
4

1. Simplifying radicals creates ______________________________________


consistency in your mathematical = 4​​ √(​    
3​​  2​∙ ​3​​ 2​∙ ​3​​ 2​∙ ​3​​ 2​) ∙ 3 ∙ (​x​​  3​∙ ​x​​  3​∙ ​x​​  3​∙ ​x​​  3​) ∙ x ​​
___
answers. When answers have a = 9​​x​​  3​​ 4​​ √3x ​​
constant form, people can easily see 27. Neither expression ____ can ___be __
and compare their results. simplified. 29. ​​  1 ​​​​√120 ​​ = ​​√30 ​​  31. 4​​√6 ​​;
__
___ 2 ___
3​​√15 ​​cannot
___ be simplified.
___ 33. 5
​ √
​ 10 ​​ 
35. ​2​ 21 ​​  37. ​4​x​​  ​y√
√ 2
​ 2y ​​  39. ​2​x​​  ​​ 
4

enVision™ Algebra 1 | 45 | Selected Answers


PearsonRealize.com

Selected Answers
Topic 9
____ ____ ____
​​ __13x ​​  43. ​5​x​​  2​​√2xy ​​  45. ​48​x​​  9√
41. ​2​x​​  7√ ​​ x ​​  Lesson 9-4

47. 4x​​ 2 ​​ft; __ 30 ft, ≈ 84.9 ft, 90 ft 1. If you have—or can simplify—a
49. a. 90​​√2 ​​ft; 127.3 ft b. 6.3 ft; The
quadratic equation so that it is in the
distance to 2nd base is
form ​a​x​​  2​ = c​, where there is no linear
127.3 – 60.5 = 66.8 ft; 66.8 – 60.5 = 6.3 ft
term, then you can divide both sides by
51. a and take the square root to solve.
√48 5x√6x3
3. If there are two solutions, and no
√12 24 30x√2x real-world context, use ±. If there
2x√6x 24x√2x 60x2 are fewer than two solutions, or if a
4x2√2x5 16x4√6x 40x4√3x solution such as a negative number
does not make sense, do not use it.
53. Part A 5. For both types of equations the
P3 P2 last two steps are to divide by a and
1 take the square root of the value.
√3
√2 P1
For equations in the form ​a​x​​  2​ ± b = c​,
√4 you first need to add or subtract__b to or
1 from c. 7. no __
solution 9. x = ±​​√6 ​​ 

3​ 10 ​ 7 ​​  15. x = ±3
O 11. x = ±​​ _____
 ​​  13. x = ±​​ __
__2 2
Part B 17. x = 5​​√2 ​​  19. The equation has no
1
1
solution; The square root of a negative
√4
√5 √3 number is undefined. 21. The student
√2
√6 1 took the square root of 4 in the next to
O √1
last step and did not include ±.
√7 √17
​−4​x​​  2​ + 19 = 3​
√8 √16 ​−4​x​​  2​ + 19 − 19 = 3 − 19​
√9 √15 ​−4​x​​  2​ = −16​
√10 √14 ​​x​​  2​ = 4​
√11 √12 √13 ​x = ±2​

Part C Unsimplified; The radicands 23. ±16 25. no solution


___
each increase by 1, but this is not 27. ±​​√91 ​​ ≈ ±9.54 29. ±5
__
apparent with simplified square roots. 31. ±10​​√10 ​​  33. ±1 35. ±5 37. ±4
__ ___
39. 0 41. ±​​√10 ​​ ≈ ±3.16 43. ​10​√10 ​ ft​;
​6x = 189.74 ft​; ​2x = 63.25 ft​ 

enVision™ Algebra 1 | 46 | Selected Answers


PearsonRealize.com

Selected Answers
Topic 9
__
45. Solve algebraically using square 19. 1 ± √ ​​ 7 ​​; −1.8 and 3.7 21. 121;
roots: ​(x + 1​1)​​  ​​  23. 6.26; ​(x − 2.​5)​​  2​​  25. 576;
2
__
​​x​​  2​ − 900 = 0​ ​(x − 24​)​​  2​​  27. ​2 ± √ ​ 34 ​​  29. −5 and 7
__
​​x​​  2​= 900​ 31. −1.5 ± √ ​​ 13.25 ​​  33. −2 and 7
___ ____

​​ ​x​​  2​ ​ = ±​√900 ​​ 35. 6.4 37. ​y = ​(x + 6)​​  2​ − 9; (−6, −9)​ 
​x = ±30​ 39. ​y = ​(x − 7)​​  2​ − 50; (7, −50)​ 
S olve using the table function on a 41. y = 2​​(x − 5)​​  2​​ − 15; (5, −15)
graphing calculator: 43. y = −4​​(x − 2)​​  2​​ + 21; (2, 21)
Enter ​y = ​x​​  2​ − 900​into a graphing 45. f(x) = −​​(x + 5)​​  2​​ + 4; yes 47. 4 m;
calculator. The vertex form of the function is
 se the table function to find the
U f(x) = ​​(x − 2)​​  2​​ + 4. So the vertex is (2, 4).
values for which ​y = 0​. This means that the lowest point of
x = ±30​
​ the dish is 4 m above the ground. The
graph of the parabola has points (0, 8)
47. a. ​−16​t​​  2​ + 67 − 3 = 0​  and (4, 8), so the width of the dish at
b. 2 s; ​−16​t​​  2​ + 67 − 3 = 0​, 8 m off the ground is 4 m. 49. 12.3 cm;
so ​−16​t​​  2​ = −64; ​t​​  2​ = 4, t = 2​  65.2 cm 51. Part A ​w = 0.5625h;
49. 48 ft 51. B A = (0.5625h + 0.4)(h + 1.2)​ 
Part B 5.23 in. Part C 5.23 in. and
Lesson 9-5 2.94 in.; 6.43 in. and 3.34 in.
1. The technique of completing the
square can be applied to any quadratic Lesson 9-6
equation with integer coefficients that 1. Use the quadratic formula when
has real number solutions. This includes you cannot factor easily and when
finding solutions that are not rational. completing the square would result in
3. The student neglected to add 16 many fractions. 3. The discriminant
to the right side, so the equation is tells you the number of roots.
no longer equivalent to the original 5. Completing the square is better than
equation. 5. It is easier to write the using the quadratic formula when
trinomial as a binomial squared when a = 1 and b is an even number, so the
the coefficient of a is 1. 7. 1 number to complete the square is a
___ ___
9. ​x = −4 ± √ ​ 15 ​​  11. ​x = 2 ± √
​ 11 ​​  positive integer. 7. ​a = −1​, ​b = 31​,
13. ​y = 5(x − 1​)​​  2​ + 2​  ​c = 7​  9. ​a = 1​, ​b = 1​, ​c = −1​  11. 0
15. a. ​​16; x​​  2​ + 8x + 16; ​(x + 4)​​  2​​  13. 2 15. The student used a plus
sign instead of a plus or minus sign. In
b. ​1​69; x​​  2​ + 26x + 169; ​(x + 13)​​  2​​ 
addition to the solution ​x = 0.39​, there
17. The student added 9 to the left side
is also the solution ​x = –3.39​.
of the equation instead of adding 18.
The correct answer is ​y = ​2(x + 3)​​  2​ − 17​.

enVision™ Algebra 1 | 47 | Selected Answers


PearsonRealize.com

Selected Answers
Topic 9

17. Shift the parabola down by 3 units. 7. ​y = 2​x​​  2​ − 5​


Then it will cross the x-axis twice, which ​y = x + 7​
means it will have 2 roots. When a 9. (0, 1) and −4, 5) 11. (−5, −6) and
quadratic function has 2 roots, the (0, −1) 13. The system has only one
discriminant is positive. 19. ​x ≈ 0.45​ solution because the graphs of the
and ​x ≈ 5.55​  21. ​x ≈ 1.24​and ​x ≈ −7.24​  two equations have only one point
23. ​x = −1​and ​x = 3.5​  25. ​x ≈ 0.28​and of intersection. 15. While using the
​x ≈ −4.73​ 27. ​x ≈ −0.88​and ​x ≈ 1.71​ elimination method, the student
29. 273; 2 31. 12; 2 33. 0; 1 subtracted an x-term from an ​​x​​  2​​-term.
35. ​−183​; 0 37. 2 real solutions; Instead of getting ​0 = −​x​​  2​ + 2​, the
greater than 0 39. The discriminant student should have gotten
must be greater than 0. The graph of ​0 = 2​x​​  2​ − 3x + 2​. Use the quadratic
the function that models the height of formula to solve. Because the
an object over time is a parabola that discriminant of the equation is negative
opens down. Because a height of 0 (​−7​) the equation and the system have
represents the ground, eventually the no solutions.
object will hit the ground. Unless the
object is launched from the ground, 17. ​y = ​x​​  2​ + 2​
one of the roots will be negative, which ​  1 ​x + 3​
​y = __
3
can be discarded in the context of the 1 ​​
19. ​x = 1 or x = −​ __
problem since time is not negative. 2
41. a. 1,250 = −5​​n​​  2​​ + 85n + 1,000 21. x ​=​2 23. ​y = 4​x​​  2​ − 2​, ​y = −​ __23 ​x + 2​;
b. ​x = 3.78, x = 13.22​; The two (−1.087, 2.725) and (0.920, 1.387)
prices that will result in the student 25. (0, 1) and (2, −13) 27. (1, 19)
council exactly meeting their goal are 29. (0, –3) and (2, –11)
$11.78 and $21.22. 43. D 31. (0.255, 14.740), and (11.745, 3.250)

Lesson 9-7 33. ​y = −16​x​​  2​ + 30​


​y = 0​; about 1.37 seconds
1. It is similar because you can use 1 ​​x​​  2​ + 2x + 10​
the same methods to solve: graphing, 35. a. ​y = −​ __
7
elimination, or substitution. It is ​ 17 ​​x​​  2​ + 2x + 10 = 2x + 3​
​y = 2x + 3​; ​−__
different because linear-quadratic
b. day 7 c. 17 people rock climbing and
systems have 0, 1, or 2 solutions. Linear
17 people zip lining 37. A
systems have 0, 1, or infinitely many
solutions. 3. It is composed of a linear Topic Review
equation and a quadratic equation,
each in two variables. 1. Check students’ work. See Teacher’s
5. ​y = ​x​​  2​ + 2x​ Edition for details. 3. completing the
​y = 3​ square 5. Product Property of Square
Roots

enVision™ Algebra 1 | 48 | Selected Answers


PearsonRealize.com

Selected Answers
Topic 9
____ ___
7. –4, 4 21. ​2√
​ 105 ​​  23. ​7x​√15 ​​  25. The__
y expressions are equal. 27. ​8√ ​ 6l ​​,
8 109.1 in./s 29. no solution 31. –0.8, 0.8
x 33. –20, 20
−5 −2.5 O 2.5 5
35. –5, 13
−8
​(x – 4​)​​  2​​ – 81 = 0
−16 ​​(x – 4)​​  2​​ = 81
_______ ___
​ √​ (x – 4​)​​  2​ ​​ = √​​ 81 ​​
x – 4 = ±9
9. no solution
y x = ±9 + 4
24 x = 13 or x = –5
37. 81, ​(x + ____ ​(x – 7.5​)​​  ​​  ___
9​)​​  2​​  39. 56.25, 2
16 ____
41. ​–9 + √ √
___​ 105 ​, –9 – ​ 105 ​​ 43. ​–11 + ​ 82 ​,

8
–11 – √ ​ 82 ​​ 45. Completing the square
x
provides an exact solution. You cannot
−5 −2.5 O 2.5 5
factor the equation, and graphing
does not provide an exact solution.
______
11. –8, 8 √
______ are 4.5 + ​​ 35.25 ​​ and
The solutions
x −8 −4 −2 0 2 4 8 4.5 – ​​ 35.25 ​​. 47. x = 2 and x = ​–__
√ ​  6 ​​
5
f(x) 0 −48 −60 −64 −60 −48 0 49. x = –0.63 and x = –2.37
51. 2 solutions 53. no solution
13. 7,000 video games; about 3,838 55. The solutions of the related
video games or about 10,162 video equation are approximately 1.87
games 15. x = –2 or x = 5 and –5.87. The negative solution is
17. x = –1.5 or x = 5 discarded because a height cannot be
19. (x – 3)(x – 5) = 0; vertex is (4, –1) negative. The only possible value is
5.87. 57. (0.5, 2) and (–1, –1) 59. (–8, 0)
y
and (–3, –5) 61. (–5, 15) 63. (3.5, 12),
(–5, –5) 65. $3
25

x
–5 O 5 10 15

enVision™ Algebra 1 | 49 | Selected Answers


PearsonRealize.com

Selected Answers
Topic 10

Lesson 10-1 29. 0.39; 3.89; 3.50


31. 1.732; 3.6056; 0.375 33. domain:
1. The graph of the square root
x ≥ 0; range: y ≥ 0 35. domain: x ≥ 3.5;
function and its translations are one
range: y ≥ 0 37. 70, 65, 74, 85
branch of a parabola with a horizontal
39. about 897 mi 41. D
axis of symmetry. The domain and
range always have a minimum value. Lesson 10-2
3. The ordered pairs have the___ values
reversed. If ​x = 12,​then ​y = √ ​ 36 ​ = 6​, so 1. domain: ​−∞ < x < ∞​; range:
(12, 6) satisfies the function. Similarly, ​−∞ < y < ∞​; increasing everywhere
___

if ​x = 27,​then ​y = ​ 81 ​ = 9​, so (27, 9) (no max/min values); intercepts axes at
satisfies the function. 5. It is a vertical (0, 0), rotational symmetry about
translation of the graph of f by 2 units the origin 3. One graph is the
down. 7. It is a vertical translation of reflection of the other across the x-axis.
the graph of f by 5 units up. 9. 0.14 5. The graphs have the same shape;
11. 0.45 13. The expression under the the graph of f is the result of shifting
radical is ​x + 3​, so the radicand has the graph of g down 3 units.
a minimum value when ​x = −3​. That 7. about 0.109 9. The graph crosses
means the x-intercept is at the point the axes at the origin, (0, 0), so the
(−3, 0) and the graph is a translation x-intercept and the y-intercept are the
__
of ​f(x) = ​√x ​​by 3 units to the left. same. 11. Hugo incorrectly factored
15. a. For f, the average rate of change out__a 3. The correct common factor
3
is 0.2; for g, the average rate of change is ​​ √3 ​​. 13. The smaller interval
is −0.2. b. The two values are opposites. provides the better approximation.
c. –0.32 17. a. The steps are to multiply 15. a. translation down 2 units
10 by 2 to get 20, find the square root b. translation right 3 units and up
of 20, and then divide that value by 7. 5 units c. translation left 1 unit
b. The steps are to multiply 10 by 2 to 17. domain: −∞ < x < ∞;
get 20, divide that value by 7, and then range: −∞ < y < ∞; x-intercept and
find the square root of the result. y-intercept: (0, 0); rotational symmetry
19. x-intercept = 9; y-intercept does not about the origin 19. domain:
exist 21. x-intercept = 77; −∞ ≤ x ≤ ∞; range: −∞ ≤ y ≤ ∞;
y-intercept −7 23. It is a horizontal x-intercept:
__ (−2, 0); y-intercept:
translation of the graph of f by 11 units (0, ​​ √ 2 ​​); rotational symmetry about
3

to the left. 25. It is a horizontal the point (−2, 0) 21. shift up


translation of the graph of f by 3 units 2 units 23. shift left 7 units
to the left and a vertical translation_____ 25. shift left __ ​​  12 ​​ unit and down ​​ __34 ​​ unit
by 6 units down. 27. g(x) = ​​√x − _​  12 ​ ​​ 

enVision™ Algebra 1 | 50 | Selected Answers


PearsonRealize.com

Selected Answers
Topic 10

27. y 7. The radicand must be nonnegative:


4
2x – 5 ≥ 0, so x ≥ ​​ __ 5 ​​ . The domain is
5 ​​, ∞). The radical2
2 [​​ __
2
indicates the
x positive root, so it will not be less than
−4 O 4 8 0. All values of f, then, must be less
−2 than or equal to 4. The range is (–∞, 4]
9. The student is distracted by the
−4
large constant. As x → ∞, the leading
3 ≤ x ≤ 10 when 1 ≤ g(x) ≤ 2 term ​​–x​​  2​​will, from some point on, be
29. 0.102 31. 0.540 33. < less than –1,000,000. Looking at the
leading term shows x → ∞, y → –∞.
35. Cars Sold 11. No, the left side of the function is
20
a mirror image of the side to the right
of the axis of symmetry. Because the
Cars Sold

15
domain is the set of all real numbers,
10 there is no vertical asymptote. Since
5 y → ∞ when x → ±∞, the graph must
0
change directions at some point,
0 1 2 3 4 5 6 7 having a minimum value. The range
Week Number
__ cannot be the set of all real numbers.
f​ (x) = 10​3√ x ​​ 13. a = –1, b = 0
Sales will continue to increase, but at a
decreasing rate. 37. a. ​​x​​  3​​ = 102,714 15. y
4
b. about 46.8 in. c. the box; about f(x) = √x − 3
2
170 ​​in.​​  2​​more surface area 39. B
x
Lesson 10-3 O 2 4 6 8 10
−2
1. the domain and range of the
function, the maximum and minimum domain: [3, ∞), range: [0, ∞)
values, asymptotes, the axis of 17. y
symmetry, and end behavior 3. Both 4 f(x) = 3 − ∣x − 4∣
types of functions change direction 2
from increasing to decreasing or from
x
decreasing to increasing. Both have −2 O 2 4 6 8
an axis of symmetry, and the end
behaviors of both graphs go in the
same direction. 5. domain: (–∞, ∞),
range: [0, ∞); minimum value is 0 when ​ domain: (–∞, ∞), range: (–∞, 3]
x = −1​; no maximum value; axis of 19. no maximum or minimum value
symmetry: ​x = −1​; as ​x → −∞​, ​y → ∞​; as ​
x → ∞​, ​y → ∞​ 

enVision™ Algebra 1 | 51 | Selected Answers


PearsonRealize.com

Selected Answers
Topic 10

21. minimum is –8 when x = 0; 7. y   


no maximum value 23. no axis 4
of symmetry
25. As x → –∞, y → ∞; as x → ∞, y → –∞. 2
27. As x → –∞, y → 0; as x → ∞, y → –∞. x
−2 O 2 4 6
29. As x → –∞, y → –∞; as x → ∞, y → –∞
−2
31. A = Miami (relatively steady high
temperatures all year)
B = Kansas City (max is highest of the 9. y    
four, min is less than Miami’s) 4
C = New York City (max/min values are 2
lower than those for KC) x
D = Anchorage (lowest max temp/ −4 −2 O 4
lowest min temp) 33. Axes of −2
symmetry will save Yumiko work
−4
because she can draw half of the image
and then reflect it. Maximum and
11. y = | x + 3 | + 2
minimum values might be helpful in
setting up edges of shapes. 35. D 13. y
g
4
Lesson 10-4
x
1. Yes; subtracting h from the function O 4 8
−4 f
input shifts the graph of the function
horizontally h units. Adding k to the
function output shifts the graph of the 15. If the graph is shifted the same
function vertically k units. 3. Ashton is integer number of units either up and
__
not correct. y = √ ​​ x ​​ has domain right or down and left, the graph will
_____
0 ≤ x < ∞. The graph of y = ​​√x – 3 ​is appear unchanged. The new function is
__
graph of y = ​​√x ​​shifted right 3 units, so f(x) = | x – a | + a, where a is an integer.
the new domain is 3 ≤ x < ∞. 17. y
8
5. y    
6
6
4
4
2
2
x
x
−4 −2 O 2 4
−4 −2 O 2 4

enVision™ Algebra 1 | 52 | Selected Answers


PearsonRealize.com

Selected Answers
Topic 10

19. y 31. The delay represents a translation


6
60 units right.
4 y
600
2
400
x
−4 −2 O 2 4 200
−2 x
0
0 20 40 60 80
21. y
8
33. A, C 35. Part A 44.6 yards
Part B 54.0 yards
4 ​​​  1  ​ ​(x − 9.4)​​  2​​ + 12
Part C g(x) = ​−____
100
2 Lesson 10-5
x
−2 O 2 4 6 1. Multiplying the output by a constant
either stretches or compresses the
23. y graph of the function vertically.
6
Multiplying the input by a constant
4 either stretches or compresses the
graph of the function horizontally.
2
3. A vertical stretch causes the graph
x
−4 −2 O 2 4
of a function to stretch away from the
−2
x-axis, and a horizontal stretch causes
the graph of a function to stretch away
25. ​y = ​(x − 1)​​  2​​  27. ​y = | x + 10 |​ from the y-axis. 5. yes 7. horizontal
compression 9. horizontal compression
29. y
8 11. 8 y
4
6
x
−8 −4 O 4 4 g
−4
2 f
−8 x
O 2 4 6 8

Based on the way the functions are


written, it is a horizontal compression.
But if you take the square root of 2
and put it in front of the radical, then
it looks like a vertical stretch.

enVision™ Algebra 1 | 53 | Selected Answers


PearsonRealize.com

Selected Answers
Topic 10
______
13. When the input is multiplied by Part B ​g(x) = √
​ 6.125 ​​f(x) Part C The
​k > 1​, it is a horizontal compression, not period is longer on the moon than on
a horizontal___ stretch. 15. ​g(x) = −​x​​  2​ + 3​ Earth for a pendulum with the same
17. ​g(x) = √ ​ 2x ​​  19. vertical stretch length. Part D The graph of g can be
21. vertical compression 23. horizontal described as either a vertical stretch
stretch 25. vertical stretch away from the x-axis or a horizontal
27. reflection across ​y = 2​; ​k = −1​  compression toward the y-axis.
__
29. g(x) = ​–​3​​√x ​​
Lesson 10-6
31. a. y
8 1. Functions can be added, subtracted,
6 and multiplied in the same way as
numbers, however, it is important to
4
apply the operation to all terms in the
2 expression. 3. Graph the combined
x function. If there are restrictions on
−4 −2 O 2 4 the x-values of the graph, then the
domain is restricted. If the graph of
The graph is compressed horizontally.
__ the function does not pass through all
b. ​s = √
​ A ​​ y-values, then the values it does not
pass through are not in the range.
c. y
8 5. (f + g)(x) = 2​x​​  2​ − x + 1
7. (f − g)(x) = 3​x​​  2​ − 2x − 7
6
9. (f ∙ g)(x) = 3​x​​  4​ − 12​x​​  3​ − 2​x​​  2​+ 8x
4 11. When adding polynomials and
functions, you combine any like terms.
2
However, functions can include terms
x
O 2 4 6 8
that cannot appear in a polynomial,
such as radicals and terms with the
d. The graph is compressed vertically. variable in the exponent. Adding
two polynomials results in another
33. ​g(x) = –| 2x + 5 |​  34. A polynomial. Adding two functions
35. Part A results in another function, but not
necessarily a type of function that
y
16 matches either of the two original
g
12
functions.

8
f
4
x
O 4 8 12 16

enVision™ Algebra 1 | 54 | Selected Answers


PearsonRealize.com

Selected Answers
Topic 10

13. y Lesson 10-7


12
1. Sometimes when solving problems,
8
f∙g f+g you know data that is the output of
4 a function. It may be easier to write
x the inverse of the function and use
−4 −2 O 2 4 the data as the input instead. 3. No;
−4 the function ​y = ​x​​  2​​does not have an
inverse function unless the domain is
15. Sample: ​f(x) = x + 3, g(x) = x + 1​, restricted.
​(f ∙ g)(x) = ​x​​  2​ + 4x + 3​. The domain and
range of f and g are all real numbers. 5.   
x y x y
The domain for f ​∙​g also includes all
0 3 3 0
real numbers, but the range changes.
The range of f ​∙​g is y ​≥​​−​1. 1 1 1 1
___ 2
17. (f + g)(x) = 2x − 3​​√x ​​ + 4 2 −1 −1
3 −3 −3 3
19. ( f − g)(x) = ​7​​  x​ − 5​x​​  2​ + 2x + 4
21. (​ f ∙ g)(x) = ​–18x​​  3​ − ​45x​​  2​ − 4x + 2​ 7. ​f​​  −1​(x) = 0.5x – 5.5​  9. The inverse of a
___ ___ function is similar to inverse operations
23. (f ∙​ ​g)(x) = 7x​​√ 5x ​​​+ 2​√ 5x ​​
because both are, in some ways, the
25. y “opposite” of the original function
8
or operation. They are different
6 because inverse functions involve
4
switching the input and output, and
inverse operations are used along
with the properties of equality to solve
x equations. 11. a. yes; ​​f​​  −1​(x) = __ ​  x5 ​​ 
−4 −2 O 2 4 b. No; restrict the domain to
The domain of ​f + g​is the same as the nonnegative numbers. c. No; restrict
domain of f and the domain of g. The the domain to nonnegative numbers.
range of ​f + g​is ​y ≥ 2​, but the range of 13. ​​f​​  −1​​(x) = 0.5(​​x​​  2​​ + 1), ​x ≥ 0​; I found
f is 4 and the range of g is ​y ≥ −2​. my answer by writing the equation
27. a. ​f(x) = 75x + 50​ b. g(x) = 36x represented by the graph and then
finding its inverse algebraically.
c. ​(f − g)(x) = 75x + 50 − 36x = 39x + 50​
29. a. f(r) = ​2πr​​  2​  b. g(r) = 40πr 15.
x y
c. (f + g)(r) = ​​2πr​​  2​ + 10πr​  31. B 11 0
15 1
19 2
23 3

enVision™ Algebra 1 | 55 | Selected Answers


PearsonRealize.com

Selected Answers
Topic 10

17. y 33.
6 _
f−1 f(x) = 4x − 8​
I. ​ A. ​ ​12 √
f(x​)​​  −1​= __ ​​ x ​
4
f(x) = 0.25x − 2​
II. ​ f(x​)​​  −1​= 0.25x + 2​
B. ​
f
x III. ​f(x) = 4​x​​  2​, x ≥ 0​ f(x​)​​  −1​= 4x + 8​
C. ​
___
−2 O 4 6
−2
IV. ​f(x) = 2​x​​  2​, x ≥ 0​ D. ​ √
​12 ​x ​
f(x​)​​  −1​= ​ __

35. Part A y = 75x + 90; y = 40x + 25


19. 10 y
​​  x –155
Part B y = 155x + 140; y = ______ 140
 ​​
8
Part C 8 months
6
f
Topic Review
4
f−1
1. Check students’ work. See Teacher’s
2
Edition for details. 3. inverse of a
x
0 function 5. The graph of g is a vertical
0 2 4 6 8 10
translation of 4 units up of the graph
21. no 23. f–1(x) = –0.2x – 2.2 of f. 7. The graph of g is a horizontal
translation of 1 unit right and a vertical
​​  x –712
25. f–1(x) = _____  ​​
____ translation of 5 units down of the
__
27. f–1(x) = √
​​ x – 7 ​​  29. y = 4x – 400; graph of f. 9. ​g(x) = ​√x ​ − 5​
$4,600 31. a. f(x) = 50 – 4x
11. y
b. f–1(x) = –0.25x + 12.5 4

c. y 2
50 x
f O 2 4 6
25 −2
f−1 x
−25 O 25 50 −4
−25
The domain is ​x ≥ 3​. The range is ​y ≤ 0​.
d. ​​f​​  −1​​;
The inverse function gives 13. The graph is a horizontal
the number of games played as a translation of 5 units left. 15. The
function of the amount of money graph is a horizontal translation of 1
left on the card. unit right and a vertical translation of
2 units up. 17. 0.09 19. 0.63, 0.18

enVision™ Algebra 1 | 56 | Selected Answers


PearsonRealize.com

Selected Answers
Topic 10

21. y 37. y
8 8 g
6 6

4 4 f

2 2
x x
−4 −2 O 2 4 −8 −4 O 4 8
The domain is the set of all real The graph of g is a vertical stretch
numbers; the range is all real numbers and a horizontal compression of
greater than or equal to 6. f. If a graph is vertically stretched,
23. As x → ∞, j(x) → −∞, and as then it is also horizontally
x → −∞, j(x) → 0. 25. f is a quadratic compressed. Conversely, if a graph is
function that opens upward. As x → ∞, vertically compressed, then it is also
f(x) → ∞, and as x → −∞, f(x) → ∞. horizontally stretched.
27. y
39. (f + g)(x) = 3​​x​​  2​​ + 7x − 8
___
12 41. (f + g)(x) = −x + √ ​​ 2x ​​ + 4
43. (f ∙ g)(x) =15​​x​​  3​​ + ​​x​​  2​​− 2x
8 __ ___
45. (f ∙ g)(x) = 6​​√3
 ​​x​​   ​​​ − 5​​√3x ​​
_ 3
2

4 47. Answers may vary. Sample:


x f(x) = 2x, g(x) = 3x, (f ∙ g)(x) = 6​​x​​  2​​
−10 −5 O 5 10
49.
x y
29. g(x) = (x – 5​​)​​ 2​​ 
31. ​g(x) = − 5​ −​x​​  2​
3 0
33. vertical stretch 35. horizontal
5 1
compression
8 2
11 3
​x​​  2​ + 3
​​  x +
51. ​​f​​  −1​​(x) = _____ 4
7
 ​​  53. f
​​ ​​  −1​​(x) = ______
​​  2
 ​​
55. The student added 3 when solving
the equation,
______
instead of subtracting 3.
√ ​  x − ​
​​f​​  −1​​(x) = ​​ _____
2
3  ​​

enVision™ Algebra 1 | 57 | Selected Answers


PearsonRealize.com

Selected Answers
Topic 11

Lesson 11-1 17. 5; A dot plot shows individual data


values.
1. dot plots, histograms, and box
plots all show the spread of a data
set. Dot plots reveal individual data
values, histograms reveal frequencies
organized by intervals, and box plots
reveal the median, quartiles, minimum, 75 77 79 81 83 85 87 89
and maximum. 3. histogram; A
histogram groups data values into 19. dot plot; You need to see individual
bins, or intervals, before displaying data values to count how many are
frequencies. 5. yes; no; yes; You can greater than any given value. 21. box
tell maximum and minimum values plot; A box plot reveals quartiles.
from a dot plot because individual 23. 7
values are displayed, and a box plot
shows maximum and minimum values. 5 7 9 11 13 15
You cannot tell in a histogram because
individual values are not displayed
within each bar.
25. 8
7. 5
7
4
6
Frequency

3
5
Frequency

2
4
1
3
0
0–2 3–5 6–8 9–11 12–14 15–17 2

1
The data are fairly evenly distributed.
0
9. box plot; 8 11. histogram; 4
9
9

9
9

9
9
.5

.9
.7
.3

.3
1.1
–0

–0
–0

–1
–0

0–

13. Answers may vary. Sample: test


20
40

80
60
20

0
1.

1.
0.

0.

0.
0.

scores for a class; Since dot plots reveal Price per ounce ($)
individual data values, you would
choose a dot plot if you wanted see Lucy’s price is lower than 70% of the
how many people got a specific test other prices.
score. 15. a. No, individual values are
not displayed in the box plot. b. No,
individual values are not displayed in
the box plot, so there is not a way to
determine the frequencies for each bin
in a histogram.

enVision™ Algebra 1 | 58 | Selected Answers


PearsonRealize.com

Selected Answers
Topic 11

27. yes; no; no; yes Lesson 11-2


29. Part A Dot plots and histograms
1. Measures of center show where a
are best for displaying the shape of
data set is clustered, while measures of
a distribution. For these data, a dot
spread show the amount of variability
plot will be too spread out, so use a
in a data set. Both types of measures
histogram.
should be considered when comparing
6
data sets. 3. Since the mean is the
5 average value, it is affected by outliers.
4 The MAD shows how closely the data
Frequency

3 are clustered about the mean, so it can


indicate the existence of values that are
2
far from the mean. 5. Data Set A: 87;
1
Data Set B: 83.2 7. Data Set A: 87;
0 Data Set B: 87 9. The ranges and IQRs
4

9
.4

9
9
9

are very close and the medians are


4.

4.
3.

3.
2.
1.

12

–1

–1

–1

–1
–1
–1

0–

.5

.5
.0

.0
.5
.5

equal, so medians, IQRs, and ranges are


.

13

13

14

14
11

12

12

Time (s)
better measures of center and spread
for comparing data sets A and B.
Part B A box plot is best for displaying 11. Since the means are the same but the
the spread of data above and below median is much smaller in one data set,
the median. that data set must include some large
values that pull the mean higher.
11.5 12.0 12.5 13.0 13.5 14.0 14.5 13. Both are measures of spread. The
IQR measures the spread of the middle
50% of data values about the median,
Part C A histogram is best for while the range measures the spread
grouping data by intervals; see Part A. of the entire data set. 15. Data Set A
Part D A dot plot is the best display to Mean: 5; MAD: 0.5; Median: 5; IQR: 1
use; to find the mean, he must know Data Set B Mean: 5; MAD: 1.375;
the individual data values. Median: 5; IQR: 2
The means are the same, but the MAD
for Data Set B is larger, so Data Set B is
11.5 12.0 12.5 13.0 13.5 14.0 14.5 more spread out around the mean.
The medians are the same, but the IQR
for Data Set B is larger, so the middle
50% of Data Set B is more spread out.
The mean and MAD are better measures;
the mean is generally preferred over the
median when both data sets are evenly
spread about the mean.

enVision™ Algebra 1 | 59 | Selected Answers


PearsonRealize.com

Selected Answers
Topic 11

17. Data Set A Mean: 74; MAD: 1.73; Lesson 11-3


Median: 74; IQR: 4; Data Set B Mean: 79;
1. The shape of a data set can help
MAD: 1.73; Median: 79; IQR: 4;
you understand relationships between
For both sets, the mean is equal to
measures of center and spread.
the median; the data are evenly
For example, when the data are
distributed. The mean and median of
symmetrically distributed, the mean
Data Set B is 5 units greater than the
and median are about the same.
mean and median of Data Set A, but
However, the mean is less than the
the MADs and IQRs are the same; the
median if the data are skewed left
sets have similar variance. The mean
and greater if the data are skewed
and MAD are better measures
right. 3. no the display for a normal
because the mean is generally
distribution is symmetric about the
preferred over the median when the
mean. 5. symmetric the mean and the
data are centered about the mean.
median are both about 12. 7. Suppose
19. The mean score on this year’s exam
two data sets have the same mean and
was 85, with a mean average deviation
median. If a much lower data value is
of 4.9. The mean score was higher
substituted for one of the data values
than last year’s score, but there was
in the lower half of the ordered set,
also greater spread about the mean
then the median will stay the same,
this year. 21. The data for the second
but the mean will decrease. 9. The
machine are not centered about the
display represents a data set that is
mean, so use the median and IQR to
skewed right. 11. a. Both displays are
compare. Machine A has a median of
symmetrical about the mean, and both
12.0, with an IQR of 0.25. Machine B
have the same spread, but Display A
has a median of 11.95, with an IQR
is 25 units to the left of Display B on
of 0.4. The median weights are either
a number line. b. Answers may vary.
equal to or very close to the advertised
Sample: If Set B did not have the same
weight, but the high variability in
shape as Set A, you could form a new
Machine B indicates that it may have a
Set B by subtracting 25 from every data
problem. 23. D
value in Set A. Then Display B would
have the same shape as Display A.
13. Both data sets are symmetric and
have the same mean and median, but
Data Set B has a greater MAD. 15. The
data display is skewed left. The mean
amount raised is less than the median.

enVision™ Algebra 1 | 60 | Selected Answers


PearsonRealize.com

Selected Answers
Topic 11

17. The display for Year 1 is skewed 7. Although the standard deviation for
right, so the mean house price was Sample A is smaller than the standard
greater than the median house price. deviation for Sample B, the mean for
The display for Year 2 is close to Sample A is also smaller than Sample B.
symmetrical, so the mean house price Therefore, you cannot use the standard
and the median house price were about deviation alone to compare the spread
the same. 19. The display for Test 1 is of one set to the other. 9. a. Since the
symmetrical, so the mean and median means of data sets are equal, you can
are close to equal. The median is about infer that the data set with the greater
525, Q1 is 400, and Q3 is 600. A student standard deviation is more spread
who takes Test 1 is likely to get a score out. b. The shape of the histogram for
between 400 and 600 and is also likely the second set is wider and flatter.
to score close to the mean score of 525. 11. Yes; 2 standard deviations below
For Test 2, the median is about 450, the mean and above the mean
Q1 is 400, and Q3 is 525. These are includes data values from about 4.28
lower than the scores for Test 1, so a to about 22. Therefore 8.7 falls within
person who takes Test 1 is more likely 2 standard deviations of the mean.
to get a higher score. Also, the display 13. Data Set A: mean: 4.25, standard
for Test 2 is skewed to the right, so the deviation: 2.49; Data Set B: mean: 5.50,
mean is greater than the median. A standard deviation: 2.67; The means
student who takes Test 2 is less likely to and the sample standard deviations
have a score close to the mean than a of both sets are close to equal, so
student who takes Test 1. 21. D the sets have similar variability.
15. Data Set A: mean: 9.06, standard
Lesson 11-4
deviation: 2.35; Data Set B: mean:
1. A data set can be analyzed by 9.06; standard deviation: 3.49; The
looking at the difference between the means are about the same, but the
greatest and least values as well as sample standard deviation for Data
patterns in how close or far data fall Set B is greater than the sample
from the mean. 3. Marisol is incorrect. standard deviation for Data Set A, so
Standard deviation measures variation the data from Data Set B have greater
from the mean. 5. Sample A has a variability. You can also verify this by
range of 4, and Sample B has a range observing that the data in the dot
of 9. So the spread of Sample B is plot for Data Set B have a greater
greater. spread. 17. 0–7 and 17–30

enVision™ Algebra 1 | 61 | Selected Answers


PearsonRealize.com

Selected Answers
Topic 11

19. Finishing times under 2:32:37 are 7.


Item A Item B Totals
more than 2 standard deviations below
Male 0.8 0.2 1.0
the mean finishing time. The histogram
bar for finishing times between 2 Female 0.4 0.6 1.0
and 3 hours represents about 25,000
runners, so there would be no more Yes; it is reasonable to conclude that
than 25,000 runners who had finishing males prefer Item A more than females
times less than 2 standard deviations do because 80% of males prefer
below the mean. 21. 700; about Item A, while only 40% of females
1,950 23. C prefer Item A.
9. Yes; an equal number of juniors and
Lesson 11-5
seniors were surveyed, so percentages
1. You can calculate joint and marginal can be compared without calculating
frequencies, joint and marginal conditional relative frequencies. The
relative frequencies, and conditional table shows that a greater percentage
relative frequencies. If conditional of juniors prefer Item B and a greater
relative frequencies in a table are percentage of seniors prefer Item A.
not all about the same, associations 11. Each marginal relative frequency is
between variables may be statistically the sum of the joint frequencies in the
significant. 3. Conditional relative corresponding row or column.
frequencies are the ratio of a joint
frequency and the marginal frequency 13.
Song A Song B Totals
of the corresponding row or column,
Teen 30 5 35
depending on which parameter, or
Adult 10 15 25
condition, you are using.
Totals 40 20 60
5.
Item A Item B Totals
Yes; the marginal frequencies show
Male 20 5 25
that more people surveyed prefer
Female 10 15 25
Song A.
Totals 30 20 50
15.
Song A Song B Totals
Teen 0.86 0.14 1.00
Adult 0.40 0.60 1.00

Yes; 86% of teens prefer Song A while


only 40% of adults do.

enVision™ Algebra 1 | 62 | Selected Answers


PearsonRealize.com

Selected Answers
Topic 11

17. High School 25. No Plant Plant


Graduate? Choice A Choice B Totals Blooms? Food Food Totals
16 96 112 < 14 5 3 8
Yes
0.08 0.48 0.56 ≥ 14 7 9 16
64 24 88
No Totals 12 12 24
0.32 0.12 0.44

Totals 80 120 200


0.40 0.60 1.00 No Plant Plant
Blooms? Food Food
< 14 0.42 0.25
19. High School ≥ 14 0.58 0.75
Graduate? Choice A Choice B Totals 1.00 1.00
Yes 0.2 0.8
No 0.8 0.2
Calculating the conditional relative
Totals 1.00 1.00 frequencies by columns shows that, of
the 12 plants grown without the new
21. No; of those who prefer Choice A, food, 58% had satisfactory blooms.
only 20% are high school graduates. Of the 12 plants grown with the new
23. the ratio of graduates who prefer food, 76% had satisfactory blooms. It
Choice B to the overall number of is reasonable to infer that there is a
respondents who chose B significant association between using
the plant food and the number of
blooms the plant will produce.
27. A, C, D
29. Part A
By how many points did your score increase?
< 50 ≥ 50 Totals
Prep
5 20 25
Course
No Prep
45 30 75
Course

Totals 50 50 100

enVision™ Algebra 1 | 63 | Selected Answers


PearsonRealize.com

Selected Answers
Topic 11

Part B No; you could calculate 13. This year’s mean is 80.2 points and
conditional relative frequencies by rows the MAD is 5.67. This year’s team had
to show that 80% of students who took a higher average score but last year’s
the prep course scored 50 points or team was more consistent in scoring
higher on the retest, compared to only near the mean. 15. skewed left
40% of students who did not take the 17. The distance from the minimum
prep course. value to the middle 50% of the data
is greater than the distance from the
By how many points did your score increase?
middle 50% to the maximum value of
< 50 ≥ 50 Totals the data. This indicates that a small
Prep value skewed the data to the left
0.20 0.80 1.00
Course
and that the mean is smaller than
No Prep
Course
0.60 0.40 1.00 the median. 19. No; 70.5 is greater
than ​50.5 + 9.6 + 9.6.​  21. more than
$24,550; 2.5% of accounts would meet
Topic Review
the requirement
1. Check students’ work. See
23.
Teacher’s Edition for details. 3. joint Where do you get most of your news?
frequencies 5. conditional relative Age TV Internet Totals
frequency 7. You would use a dot plot ≤30 25% 40% 65%
when you want to know individual >30 15% 20% 35%
data values or see clusters, gaps, and Totals 40% 60% 100%
outliers. 9. You would use a histogram
because it shows the frequency of data No, most people from each age
values within an interval. 11. The data group prefer to get their news from
are skewed right. the Internet compared to TV
(60% vs. 40%).

enVision™ Algebra 1 | 64 | Selected Answers

You might also like